Remember me
▼ Content

Do I have the CO2 calamity math right? (help from an expert please)



Page 3 of 6<12345>>>
30-09-2019 10:44
Into the NightProfile picture★★★★★
(21596)
VernerHornung wrote:
tmiddles wrote:
So the asci format is a bit tough with fractions like this.

Ja. Typing equations here sucks.
~


tmiddles wrote:
So frequency is a huge number and wavelength a tiny number. f is in the numerator and λ in the denominator so they are achieving the same effect of imbiggening the number (science lingo).

Ja. If the frequency goes up or the wavelength gets smaller, the fraction in the equation grows larger. Numbers get unwieldy in Planck's law. The Planck constant,

h = 0.00000 00000 00000 00000 00000 00000 000667 Joule seconds

is usually written as 6.67 x 10^-34 so they don't have to mess with 33 zeros after the decimal point. The smallness of h is why people don't notice quantum effects in daily life; mostly it affects atoms or molecules.

The units on your equations are correct, and they matter. The square meters and π tell us it's for energy radiated over a whole sphere. As for why f is upstairs to the 3rd power and λ downstairs to the 5th power, I think I have a clue. The Phys. Prof. had two of us young'uns hold the ends of a jump rope and excite it into a standing wave. Pair with the most nodes won, and nobody achieved more than three since you have jiggle the rope faster—the frequency—to get a higher number of nodes.

tmiddles wrote:
...but not to the λ^5th! arggh! So close.

Getting the exponents is tough; it took me a while to arrive at a schema for it. Light was still viewed as a wave in Planck's day. So he proposed the radiation in a cavity radiator was a standing wave; it's convenient to assume a cubical shape. Let's start with a one-dimensional standing wave confined between walls, like this:



The following derivation's extreme slop (glad there's no real physicists here), but it's meant for concept only. The energy in a classical standing wave is proportional to the amplitude (how tall the up and down swings are) and to the number of nodes, so we model the cavity radiator as a hollow box with standing waves inside:



In the last part, I'll just put in Einstein's photon energy to get numbers that correspond to the energy of the standing waves near a particular frequency or wavelength:



This isn't the correct derivation; it just explains the exponents on f and λ. The numeral 3 emerged because I used a box. When a cubical box of L x L x L inches expands a small amount ΔL, the increase in volume is about 3L^2 ΔL. We can check that. Suppose the box is 30 inches on a side and expands by ΔL of 1 inch. Then

initial volume: 27000 in^3
estimated increase: 3 x (30 in)^2 x 1 in = 2700 in^3
estimated final volume: 29700 in^3

exact final volume: (31 in)^3 = 29791 in^3

Pretty close, and the approximation gets better the smaller ΔL is. We speak of an "instantaneous rate of volume increase" for boxes,

dV = 3L^2 dL,

which is 2700 cubic inches per inch for a 30-inch box. The shape of the cavity doesn't matter very much, and we're interested in the rays outside the cavity, the ones that come out through the little hole. For that I'll need to derive the stuff I have above for a spherical shell, like the skin of a basketball. The numeral 3 should disappear, with 4π in its place instead.

I'm slow, though, and they didn't spend much time on Planck's law in school. I'd forgotten about it until now. Max was a smart cookie.

The whole point of the exercise is the "Stefan-Boltzmann" scream coming from that pair of ITN-IBdaMann speakers on the bookcase. With CO2, we have something basically like a Planck spectrum with a slot cut out in the infrared, like



The sunlight comes down with most of its energy in the slot where CO2 is transparent. Rays from the Earth come up near the right end where CO2 absorbs. The asymmetry is crucial to making the greenhouse work; otherwise the CO2 would just be blocking solar energy before it got to the surface.


Misuse of Planck's law. Denying the Stefan-Boltzmann law. CO2 also radiates.

Absorption of surface IR does not warm the Earth.

* You can't trap or slow heat.
* You can't trap light.
* You can't trap thermal energy. There is always heat.
* You can't reduce the radiance of Earth and increase it's temperature.


The Parrot Killer

Debunked in my sig. - tmiddles

Google keeps track of paranoid talk and i'm not on their list. I've been evaluated and certified. - keepit

nuclear powered ships do not require nuclear fuel. - Swan

While it is true that fossils do not burn it is also true that fossil fuels burn very well - Swan
01-10-2019 12:18
tmiddlesProfile picture★★★★★
(3979)
VernerHornung wrote:Smith answers that "the average temperature is always less than or equal to the effective thermal radiation temperature" computed by Gerlich. Head-scratch time! The papers are at
tmiddles wrote:
Are there filters, like you'd put on a light or camera lens, that have this same property of transmitting high frequency and absorbing low frequency?

There are various filters for telescopes & cameras.


The math is beyond me at the moment (I might be able to get up to speed in time). We are lucky to have REAL other planets to reference in testing these theories.

What I'm wondering is why you can't have a light shining on a box covered with a filter that allows high frequency light in but absorbs infra red. It seems that if that if you had enough of this type of filtering you could have an extreme greenhouse effect.

"Good tests kill flawed theories; we remain alive to guess again." - Karl Popper
ITN/IBD Fraud exposed:  The 2nd LTD add on claiming radiance from cooler bodies can't be absorbed Max Planck debunks, they can't explain:net-thermal-radiation-you-in-a-room-as-a-reference & Proof: no data is ever valid for them
01-10-2019 19:23
Into the NightProfile picture★★★★★
(21596)
tmiddles wrote:
VernerHornung wrote:Smith answers that "the average temperature is always less than or equal to the effective thermal radiation temperature" computed by Gerlich. Head-scratch time! The papers are at
tmiddles wrote:
Are there filters, like you'd put on a light or camera lens, that have this same property of transmitting high frequency and absorbing low frequency?

There are various filters for telescopes & cameras.


The math is beyond me at the moment (I might be able to get up to speed in time). We are lucky to have REAL other planets to reference in testing these theories.

What I'm wondering is why you can't have a light shining on a box covered with a filter that allows high frequency light in but absorbs infra red. It seems that if that if you had enough of this type of filtering you could have an extreme greenhouse effect.


* There is no frequency term in the Stefan-Boltzmann law.

Absorption of visible light doesn't convert to thermal energy. Absorption of infrared light does. The filter will get warm, the box will not.


The Parrot Killer

Debunked in my sig. - tmiddles

Google keeps track of paranoid talk and i'm not on their list. I've been evaluated and certified. - keepit

nuclear powered ships do not require nuclear fuel. - Swan

While it is true that fossils do not burn it is also true that fossil fuels burn very well - Swan
02-10-2019 03:52
VernerHornungProfile picture★☆☆☆☆
(133)
tmiddles wrote:
The math is beyond me at the moment (I might be able to get up to speed in time).

To clear my record, I should reference the steps physicists take in deriving Planck's law, up to where I was at in my little gray boxes. They figure the number of states in a box as I did, but for the frequencies they choose a sphere inside the box, of radius proportional to



because the waves actually radiate in all directions inside the cavity, although they still have nodes at the walls. This produces the factor 8π/c^3 instead of just 3 as I had above. You may skip it if desired. (Ever notice how the Man and the Parrot Killer keep trashing my art? Shame.)
~


And we're nowhere near finished walking the Planck yet! A careful, complete derivation is in the link here. Following it, I'm now at the middle of p. 5, where equation 10.5 gives the number of states to use. The equation isn't for a whole volume, however, just for a thin spherical "shell" of frequencies between f and f + Δf, where Δf is much smaller than f. The construction imposes a few conditions on use of Planck and Stefan-Boltzmann I'll tell the Parrot when I get to him below.

Derivation of the Planck Formula
University of Sao Paulo, Brazil
https://edisciplinas.usp.br/pluginfile.php/48089/course/section/16461/qsp_chapter10-plank.pdf

tmiddles wrote:
We are lucky to have REAL other planets to reference in testing these theories.

Ja. I think Mars is a better test case, though. The surface is visible from space, with full daylight sun. It has seasons like Earth's and a similar rotation period. It has a similar amount of total greenhouse gases because its air, though thin, is all CO2. Yet it has less of a greenhouse effect than Earth does. That's partly the weaker sun at its orbit, and because it has no significant water vapor.

tmiddles wrote:
What I'm wondering is why you can't have a light shining on a box covered with a filter that allows high frequency light in but absorbs infra red.

I think it's already been done, and put on the market: those newfangled heat-loss prevention windows they sell for houses. Lets daylight in, keeps infrared inside the house from radiating back out the windows, a major source of heat loss in winter.

A house is heated by its furnace, a planet by its star. The basic physics of greenhouse effects looks good to me, but applying it to planets with dynamic atmospheres is difficult and that's what makes me skeptical of getting precise values for surface temperature rise as you add CO2 to Earth's atmosphere in a computer model. The more elaborate the model, the more chances for hidden mistakes or bugs, and planets may have climate drivers we don't even know about yet.

Into the Night wrote:
Misuse of Planck's law. Denying the Stefan-Boltzmann law. CO2 also radiates.

Stefan-Boltzmann is based on Planck, what you get if you integrate the emission curve over all frequencies (or, equivalently, all wavelengths) from 0 to ∞. Planck, in turn is based on several assumptions, the key one here being that the dimensions of the cavity radiator generating the waves is much larger than the wavelengths. Planck works for continuous thermal emission from blackbodies but cannot be applied to the spectral lines of atoms or molecules because these objects are smaller than the wavelengths of their spectral lines.

As consequence, Stefan-Boltzmann doesn't work for filtered spectra as often seen with gases in quite the same way it does for opaque, colored solid surfaces. Gases and certain transparent materials such as glass are forced to radiate in frequency ranges available to them. I never said CO2 doesn't radiate. But it doesn't absorb or radiate much at visible wavelengths.

Into the Night wrote:
You can't trap or slow heat.

I'll keep that in mind every time I have to put my coat on this winter.
~



Never try to solve an NP-complete problem on your own with pencil & paper.
02-10-2019 07:31
Into the NightProfile picture★★★★★
(21596)
VernerHornung wrote:
tmiddles wrote:
What I'm wondering is why you can't have a light shining on a box covered with a filter that allows high frequency light in but absorbs infra red.

I think it's already been done, and put on the market: those newfangled heat-loss prevention windows they sell for houses. Lets daylight in, keeps infrared inside the house from radiating back out the windows, a major source of heat loss in winter.

Nope. It's just two panels of glass with an inert gas sealed within to prevent fogging between them.
Absorption of visible light doesn't convert to thermal energy. It converts to chemical energy.
VernerHornung wrote:
A house is heated by its furnace, a planet by its star.

Brilliant. Took you that long to figure it out, eh?
VernerHornung wrote:
The basic physics of greenhouse effects looks good to me,

* You cannot create energy out of nothing.
VernerHornung wrote:
but applying it to planets with dynamic atmospheres is difficult and that's what makes me skeptical of getting precise values for surface temperature rise as you add CO2 to Earth's atmosphere in a computer model.

Computer models are not data. You can't create energy out of nothing.
VernerHornung wrote:
The more elaborate the model, the more chances for hidden mistakes or bugs, and planets may have climate drivers we don't even know about yet.

Climate doesn't have drivers. You can't create energy out of nothing.
VernerHornung wrote:
Into the Night wrote:
Misuse of Planck's law. Denying the Stefan-Boltzmann law. CO2 also radiates.

Stefan-Boltzmann is based on Planck, what you get if you integrate the emission curve over all frequencies (or, equivalently, all wavelengths) from 0 to ∞. Planck, in turn is based on several assumptions, the key one here being that the dimensions of the cavity radiator generating the waves is much larger than the wavelengths. Planck works for continuous thermal emission from blackbodies but cannot be applied to the spectral lines of atoms or molecules because these objects are smaller than the wavelengths of their spectral lines.

While the Stefan-Boltzmann law can be derived from Planck's law, they are not the same. You are attempting to introduce a frequency term in the Stefan-Boltzmann law. You are denying science again.
VernerHornung wrote:
As consequence, Stefan-Boltzmann doesn't work for filtered spectra as often seen with gases in quite the same way it does for opaque, colored solid surfaces.

It works for ALL bodies, including gases. There is no term for state of matter in the Stefan-Boltzmann law. There is no term for surface color in the Stefan-Boltzmann law.
VernerHornung wrote:
Gases and certain transparent materials such as glass are forced to radiate in frequency ranges available to them.

No. They all radiate according to the Stefan-Boltzmann law. Nothing is forced to radiate 'on frequency'.
VernerHornung wrote:
I never said CO2 doesn't radiate. But it doesn't absorb or radiate much at visible wavelengths.

1) The Earth emits all the energy absorbed from the Sun.
2) The Earth does not emit all the energy absorbed from the Sun.
Irrational. Which is it dude?

1) Three photons go up, only two make to space.
2) Two photons go up, they both make to space.
Which is it dude?

VernerHornung wrote:
Into the Night wrote:
You can't trap or slow heat.

I'll keep that in mind every time I have to put my coat on this winter.

Coats and blankets do not trap or slow heat. They reduce heat.


The Parrot Killer

Debunked in my sig. - tmiddles

Google keeps track of paranoid talk and i'm not on their list. I've been evaluated and certified. - keepit

nuclear powered ships do not require nuclear fuel. - Swan

While it is true that fossils do not burn it is also true that fossil fuels burn very well - Swan
02-10-2019 18:47
IBdaMannProfile picture★★★★★
(14404)
VernerHornung wrote: Planck works for continuous thermal emission from blackbodies but cannot be applied to the spectral lines of atoms or molecules because these objects are smaller than the wavelengths of their spectral lines.

This is a stupidly absurd assertion. There is no thermal radiation emitted by any body that isn't coming from its molecules. There are no molecules that aren't radiating as bodies.

VernerHornung wrote: As consequence, Stefan-Boltzmann doesn't work for filtered spectra as often seen with gases in quite the same way it does for opaque, colored solid surfaces.

Stefan-Boltzmann applies to all matter therefore it applies to all molecules.

Find one molecule for which it does not work.



VernerHornung wrote:
Into the Night wrote:
You can't trap or slow heat.

I'll keep that in mind every time I have to put my coat on this winter.
~

... and you'll keep that in mind whenever you fail to understand the meaning of the words you misuse.

You can't "trap" heat. If you think you can, you don't know what it means.

.


I don't think i can [define it]. I just kind of get a feel for the phrase. - keepit

A Spaghetti strainer with the faucet running, retains water- tmiddles

Clouds don't trap heat. Clouds block cold. - Spongy Iris

Printing dollars to pay debt doesn't increase the number of dollars. - keepit

If Venus were a black body it would have a much much lower temperature than what we found there.- tmiddles

Ah the "Valid Data" myth of ITN/IBD. - tmiddles

Ceist - I couldn't agree with you more. But when money and religion are involved, and there are people who value them above all else, then the lies begin. - trafn

You are completely misunderstanding their use of the word "accumulation"! - Climate Scientist.

The Stefan-Boltzman equation doesn't come up with the correct temperature if greenhouse gases are not considered - Hank

:*sigh* Not the "raw data" crap. - Leafsdude

IB STILL hasn't explained what Planck's Law means. Just more hand waving that it applies to everything and more asserting that the greenhouse effect 'violates' it.- Ceist
02-10-2019 18:48
VernerHornungProfile picture★☆☆☆☆
(133)
Into the Night wrote:
You cannot create energy out of nothing.

The energy comes from the sun. It's not being created from nothing. Greenhouse gases slow the rate of heat loss from a planet's surface, but would not stop the planet from freezing to near absolute zero were the sun to stop shining.

Into the Night wrote:
1) The Earth emits all the energy absorbed from the Sun.
2) The Earth does not emit all the energy absorbed from the Sun.
Irrational. Which is it dude?

1) Three photons go up, only two make to space.
2) Two photons go up, they both make to space.
Which is it dude?

The in-out balance is preserved. Two photons arrive from the sun and the Earth surface emits three, one of which is returned to the surface by the atmosphere. The other two photons exit to outer space. The surface is able to emit three because it's hotter than it would be without an atmosphere above it. Conservation of energy is obeyed at the top of the atmosphere, two in, two out, and at the surface, three in, three out. No energy is created because one of the three photons involved is ping-ponging back and forth between surface and atmosphere.

If you add more gas, the surface will then warm to a still-higher temp. But it will do so only if the sun shines on it. If you switched the sun off and added gas, the surface would of course chill, exactly as I will if I happen to die with my coat on!

What I don't get here, ITN, is some of your posts from past years were quite reasonable, such as this one, which said that ocean currents rotate clockwise and tides are caused by the moon and sun, which pull harder on the side of the Earth closest to them. You accurately countered the "whirlpools" absurdity the top post in the thread had expressed.

Tides is the result of the rotation of the Earth and whirlpools
https://www.climate-debate.com/forum/tides-is-the-result-of-the-rotation-of-the-earth-and-whirlpools--d18-e1744.php#post_30161

Lately, we notice weird acronyms, private vocabulary, and repetitive mantras. What's changed? If you can state in clear, unambiguous terms why an atmospheric greenhouse is impossible, or link back to a post where you've done so, then I'll be willing to look. I'm not a pretend physics expert here; there's a lot I don't understand.

Raymond Pierrehumbert, Arthur Smith, Carl Sagan and other scientists have established, however, that atmospheric greenhouses obey energy conservation. Physicists who object to the greenhouse instead raise other concerns, for Gerlach mainly that

1) the formulas of cavity radiation are used inappropriately
2) thermal conductivity and friction [in the atmosphere] must not be set to zero,

where doing the latter allows the air to run as a kind of perpetual motion machine, like a wheel that spins forever on a frictionless axle. These objections are at least serious. While 97% in the geophysics, meteorology and climatology communities are said to concur with the greenhouse model, the existence of a 3% dissent should not be dismissed. If the model were as obvious as E = 0.5kA^2 for a harmonic oscillator at amplitude A, there'd be no dissent at all.


Never try to solve an NP-complete problem on your own with pencil & paper.
Edited on 02-10-2019 18:51
02-10-2019 19:19
IBdaMannProfile picture★★★★★
(14404)
VernerHornung wrote:
Into the Night wrote:
You cannot create energy out of nothing.

The energy comes from the sun.

Then you are shifting semantic goalposts and are violating Stefan-Boltzman. You are claiming a reduction in radiance with an increase in temperature. Temperature and Radiance move in the same direction.

VernerHornung wrote: It's not being created from nothing. Greenhouse gases slow the rate of heat loss from a planet's surface,

Right here. You are declaring a reduction in Radiance. Nevermind that you are using entirely the wrong wording, that you have no clue what "heat" is, that you don't know what you mean by "slowing" heat.

You are claiming that Radiance is reduced.

VernerHornung wrote:If you add more gas, the surface will then warm to a still-higher temp.

Right here you are declaring an increased temperature.

You are violating Stefan-Boltzmann. Period. Your argument is dismissed.

VernerHornung wrote: The in-out balance is preserved.

Nope.
Given the earth, the same number of photons must be emitted as are absorbed. Your attempt to circumvent this by subdividing the atomic unit renders your argument null and void.

Try again when your photons add up. Dismissed.

VernerHornung wrote:What I don't get here, ITN, is some of your posts from past years were quite reasonable, ...
Lately, we notice weird acronyms, private vocabulary, and repetitive mantras.

The problem is on your end. You are asking what has changed with your arrival, yes? The answer would be your arrival. The problem must lie therein, yes?

But thank you for clarifying that you and tmiddles have been collaborating and that neither of you have any excuse for not knowing what was explained in the past except for you having ignored it because you smelled an overpoweringly repulsive odor of disagreement.

VernerHornung wrote:Raymond Pierrehumbert, Arthur Smith, Carl Sagan and other scientists have established, however, that atmospheric greenhouses obey energy conservation.

No, they have not ... which means you have not established this, and pretending to speak for other people who are not present to be cross-examined is not a trump card that you get to play.

VernerHornung wrote: While 97% in the geophysics, meteorology and climatology communities are said to concur with the greenhouse model,

Incorrect.

1) there is no model that doesn't violate physics
2) only political activists believe in violations of physics

VernerHornung wrote: the existence of a 3% dissent should not be dismissed.

All arguments claiming that science is somehow based on consensus and on democratic vote as if some group, organization or institution somehow owns science, are summarily dismissed.


.


I don't think i can [define it]. I just kind of get a feel for the phrase. - keepit

A Spaghetti strainer with the faucet running, retains water- tmiddles

Clouds don't trap heat. Clouds block cold. - Spongy Iris

Printing dollars to pay debt doesn't increase the number of dollars. - keepit

If Venus were a black body it would have a much much lower temperature than what we found there.- tmiddles

Ah the "Valid Data" myth of ITN/IBD. - tmiddles

Ceist - I couldn't agree with you more. But when money and religion are involved, and there are people who value them above all else, then the lies begin. - trafn

You are completely misunderstanding their use of the word "accumulation"! - Climate Scientist.

The Stefan-Boltzman equation doesn't come up with the correct temperature if greenhouse gases are not considered - Hank

:*sigh* Not the "raw data" crap. - Leafsdude

IB STILL hasn't explained what Planck's Law means. Just more hand waving that it applies to everything and more asserting that the greenhouse effect 'violates' it.- Ceist
02-10-2019 22:21
Into the NightProfile picture★★★★★
(21596)
VernerHornung wrote:
[quote]Into the Night wrote:
You cannot create energy out of nothing.

The energy comes from the sun.
Irrational. You are locked in paradox.
1) The energy comes from the Sun. No energy is created.
2) The energy comes from CO2.
Which is it, dude?
VernerHornung wrote:
It's not being created from nothing. Greenhouse gases slow the rate of heat loss from a planet's surface, but would not stop the planet from freezing to near absolute zero were the sun to stop shining.

Irrational. You MUST clear your paradox.
VernerHornung wrote:
[quote]Into the Night wrote:
1) The Earth emits all the energy absorbed from the Sun.
2) The Earth does not emit all the energy absorbed from the Sun.
Irrational. Which is it dude?

1) Three photons go up, only two make to space.
2) Two photons go up, they both make to space.
Which is it dude?

The in-out balance is preserved.
VernerHornung wrote:
Two photons arrive from the sun and the Earth surface emits three, one of which is returned to the surface by the atmosphere.

* You can't create energy out of nothing.
VernerHornung wrote:
The other two photons exit to outer space. The surface is able to emit three because it's hotter than it would be without an atmosphere above it.

* You can't create energy out of nothing.
* Temperatures measured on the Moon's daytime side are MUCH higher than Earth, the same distance from the Sun. The atmosphere is not warming the Earth.
VernerHornung wrote:
Conservation of energy is obeyed at the top of the atmosphere, two in, two out, and at the surface, three in, three out. No energy is created because one of the three photons involved is ping-ponging back and forth between surface and atmosphere.
* You can't create energy out of nothing. You can't suspend the 1st law of thermodynamics at the surface.
[quote]VernerHornung wrote:
If you add more gas, the surface will then warm to a still-higher temp.

* You can't create energy out of nothing.
VernerHornung wrote:
But it will do so only if the sun shines on it.

The Sun is putting out the same energy.
VernerHornung wrote:
If you switched the sun off and added gas, the surface would of course chill, exactly as I will if I happen to die with my coat on!

Irrational. Which is it, dude?
VernerHornung wrote:
What I don't get here, ITN, is some of your posts from past years were quite reasonable, such as this one, which said that ocean currents rotate clockwise and tides are caused by the moon and sun, which pull harder on the side of the Earth closest to them. You accurately countered the "whirlpools" absurdity the top post in the thread had expressed.
They are still reasonable. I have done nothing more than remind you of the theories of science you continue to ignore.
VernerHornung wrote:
Lately, we notice weird acronyms,

I have explained every acronym. Argument of the stone fallacy.
VernerHornung wrote:
private vocabulary,

The 1st and 2nd laws of thermodynamics and the Stefan-Boltzmann law is not a 'private vocabulary'.
VernerHornung wrote:
and repetitive mantras.

The 1st and 2nd laws of thermodynamics and the Stefan-Boltzmann law does not change. You have also not changed in the way you ignore these laws.
VernerHornung wrote:
What's changed?

Nothing.
VernerHornung wrote:
If you can state in clear, unambiguous terms why an atmospheric greenhouse is impossible,

Already did. Multiple times. RQAA.
VernerHornung wrote:
or link back to a post where you've done so, then I'll be willing to look.

No. I don't link back. You have to go find it yourself. Pay attention next time.
VernerHornung wrote:
I'm not a pretend physics expert here;

Yes you are liar.
VernerHornung wrote:
there's a lot I don't understand.

That's obvious.
VernerHornung wrote:
Raymond Pierrehumbert, Arthur Smith, Carl Sagan and other scientists have established, however, that atmospheric greenhouses obey energy conservation.

There is no such thing as a 'greenhouse' gas. No gas or vapor is capable of warming the Earth using IR emitted from Earth's surface.
VernerHornung wrote:
...deleted irrelevant material...
[quote]VernerHornung wrote:
These objections are at least serious. While 97% in the geophysics, meteorology and climatology communities are said to concur with the greenhouse model, the existence of a 3% dissent should not be dismissed.


Dismissed.

Argument from randU. Math error: failure to show raw data. Failure to use unbiased raw data. Failure to select by randN. Failure to normalize by paired randR. Failure to declare variance. Failure to calculate margin of error.

Consensus is not used in science. It is only used in religions and governments.
* There is no voting bloc in science.
* No theory is ever proven True, either in science or out of it.
Science is not a 'community'. It is not a political organization. It does not run by votes. No theory is created by vote. No theory is tested by vote.
VernerHornung wrote:
If the model were as obvious as E = 0.5kA^2 for a harmonic oscillator at amplitude A, there'd be no dissent at all.

This is not a model. It's a random equation. The oscillator is not specified. Void argument fallacy. Mathematics is not subject to voting either.


The Parrot Killer

Debunked in my sig. - tmiddles

Google keeps track of paranoid talk and i'm not on their list. I've been evaluated and certified. - keepit

nuclear powered ships do not require nuclear fuel. - Swan

While it is true that fossils do not burn it is also true that fossil fuels burn very well - Swan
Edited on 02-10-2019 22:22
03-10-2019 04:49
VernerHornungProfile picture★☆☆☆☆
(133)
IBdaMann wrote:
Then you are shifting semantic goalposts and are violating Stefan-Boltzman.

No moving goalposts here. I've stated over and over again that the energy comes from the sun. And no violation of Stefan-Boltzmann because the Earth isn't a single radiating surface. It is a composite system with parts that radiate separately according to their own temperatures and emissivity characteristics. This isn't something I invented on my own; it's been developed by chemists and physicists since Svante Arennhius (1896), becoming a matter of interest to planetary astronomers long before the political controversies over carbon arose. Meaning it wasn't invented to support the climate change agenda, either.

There may well be other problems with the atmospheric greenhouse theory, but ex nihilo energy creation and Stefan-Boltzmann are not among them.


Never try to solve an NP-complete problem on your own with pencil & paper.
Edited on 03-10-2019 04:54
03-10-2019 04:53
tmiddlesProfile picture★★★★★
(3979)
Into the Night wrote:
Absorption of visible light doesn't convert to thermal energy.
So what does it convert to?

VernerHornung wrote:...Ever notice how the Man and the Parrot Killer keep trashing my art?...
Inevitable result of the law: "Those who can, do. Those who can't, criticize."

VernerHornung wrote:
Ja. I think Mars is a better test case, though. The surface is visible from space, with full daylight sun. It has seasons like Earth's and a similar rotation period. It has a similar amount of total greenhouse gases because its air, though thin, is all CO2. Yet it has less of a greenhouse effect than Earth does. That's partly the weaker sun at its orbit, and because it has no significant water vapor.
So we have Mercury, with no atmosphere, giving us 4 degrees more than what we'd expect for the distance from the Sun. Venus is crazy town. Earth is 30 degrees above and Mars is just 6 degrees above.
Really with Mercury coming in at 4 degrees above and what has to be a healthy margin of error Mars isn't really doing ANYTHING in observed vs. calculated.

Earth is at 1013.25 millibars and Mars just 6.0 millibars pressure at ground level. That's just 0.6%. While CO2 is 0.04%, so Mars has 15 times the CO2. BUT the water vapor is 0-4% so that is overwhelming all told right? Really Mars makes a very solid case for Huffman's point, maybe more than Venus does.

VernerHornung wrote:
applying it to planets with dynamic atmospheres is difficult and that's what makes me skeptical of getting precise values for surface temperature rise as you add CO2 to Earth's atmosphere in a computer model.
Yes and any theory being modeled should also get the other planets right as well.

VernerHornung wrote:
Into the Night wrote:
You can't trap or slow heat.

I'll keep that in mind every time I have to put my coat on this winter.~
ITN likes to be deceptive with vocabulary games. I think he'd say you can "reduce heat". But it's all a feverish attempt to claim thermal energy cannot build up anywhere like we warmazombies so passionately want it to.

Remember that in IBD/ITN land that coat won't help you. You're a goner in even a 70F room!

"Good tests kill flawed theories; we remain alive to guess again." - Karl Popper
ITN/IBD Fraud exposed:  The 2nd LTD add on claiming radiance from cooler bodies can't be absorbed Max Planck debunks, they can't explain:net-thermal-radiation-you-in-a-room-as-a-reference & Proof: no data is ever valid for them
03-10-2019 07:48
VernerHornungProfile picture★☆☆☆☆
(133)
tmiddles wrote:
Inevitable result of the law: "Those who can, do. Those who can't, criticize."

I'm in fact done with energy and Stefan-Boltzmann, satisfied what whatever might be wrong with the greenhouse doesn't involve these. Time to move on and find out what it is.

tmiddles wrote:
Earth is at 1013.25 millibars and Mars just 6.0 millibars pressure at ground level. That's just 0.6%. While CO2 is 0.04%, so Mars has 15 times the CO2. BUT the water vapor is 0-4% so that is overwhelming all told right? Really Mars makes a very solid case for Huffman's point, maybe more than Venus does.

The solar flux at 1 AU (Earth orbit) is 1360 W/m^2, so at 1.52 AU Mars will be getting 1360/1.52^2, or 589 Watts per square meter, just above the atmosphere and where the sun is directly overhead. Averaging over a sphere gives one-fourth this value, or 147 Watts, versus the 340 Watts the IPCC reports for each square meter on Earth. We can easily compute the blackbody temps:

T = (R/σ)^(1/4), with σ = 5.67 x 10^(-8) W / m^2 K^4

Earth: (340/5.67 x 10^(-8))^1/4 = 278K
Mars: (147/5.67 x 10^(-8)))^1/4 = 226K

So we have our first deviation from the blackbody situation: the albedo of each planet. On Earth, 30% of the R is reflected away without heating the surface, on Mars, 25%. So we redo the calculation with an adjustment for this:

Earth: (0.70 x 340/5.67 x 10^(-8))^1/4 = 255K
Mars: (0.75 x 147/5.67 x 10^(-8)))^1/4 = 210K

These match the "predicted temperatures" in the planet chart within a margin of rounding error. Earlier on the forum, I figured the greenhouse gases per square meter by mass, but I need to refine the formula a bit to account for CO2 being heavier than air and H2O lighter:

mass(CO2) = partial pressure (CO2) x molar mass (CO2) divided by molar mass (air ) divided by surface acceleration of gravity

Earth pressure 101.3 kPa, 0.04% CO2, 1.4% H2O
partial pressure of CO2 is 0.0004 x 101.3 kPa = 41 Pa
molar mass of CO2 = 44
molar mass of air = 29
gravity = 9.8 m/s^2
so mass (CO2) = (41 x 44) /(29 x 9.8) = 6.3 kg of CO2 per square meter

partial pressure of H2O is 0.014 x 101.3 kPa = 1420 Pa
molar mass of H2O = 18
so mass (H2O) = (1420 x 18) / (29 x 9.8) = 90 kg of H2O per square meter

Mars pressure 640 Pa, 95% CO2, 0.03% H2O
partial pressure of CO2 is 0.95 x 640 Pa = 608 Pa
molar mass of Martian air = 43.5
gravity = 3.7 m/s^2
so mass (CO2) = (608 x 44)/(43.5 x 3.7) = 166 kg of CO2 per square meter.

partial pressure of H2O is 0.0003 x 640 Pa = 0.19 Pa
so mass (H2O) = (0.19 x 18) / (43.5 x 3.7) = 0.021 kg of H2O per square meter



Mars has more greenhouse gases total overhead, per unit of surface area, than Earth does! These computations are rough because gravity decreases with height and other little things. Convection is also stronger, in relative terms, on Mars than on Earth because the air is very dry and nearly cloudless. (The clouds you see in the diagram are thin and there aren't many visible in rover photos.) the tropopause on Mars is over 30 km above ground, though the thinness of the air means convection doesn't carry much heat up from the surface.

But remember the pink sky of Viking? Mars has a load of junk in the air—no rain washing it out, so the Huffman-style direct absorption of sunlight by particles in the air will be important on Mars as well, if less so than on Venus & Earth. Global dust storms are known to drop surface temps by blocking sun; the Opportunity rover survived and took thermometer measurements during one. They heat the atmosphere aloft, however, as measured by the orbiters.

I dunno. I doubt direct heating of the air by sunlight produces the greenhouse effect itself, but it tends to decrease the temperature difference between air & ground, reducing convection. (A hot air balloon rises faster on a cold day.) Reduced convection, in turn, means a warmer surface because less heat from the ground is carried upward. The heating of air by sun warms the surface in an indirect way, and should do so without any greenhouse gases present.

That's my take on it, which is probably leaky as a sieve. Weather & climate is a complicated mess. Nobody in climate change has ever told us why the Martian "greenhouse" is only 6˚C with more gases than we have, and Earth's is 33˚. Reason could be the weaker sunlight there, or the optical thickness parameter in spectrography. (The Earth's CO2, despite its tiny concentration, is enough to block any 15-μm ray from reaching space from the ground, and such rays are also blocked on Mars.)

Seems greenhouse absorption/re-emission spectra, direct heating of air & clouds, vertical convection and horizontal winds all must be analyzed to compute a surface temp. It's beyond my ability.


Never try to solve an NP-complete problem on your own with pencil & paper.
Edited on 03-10-2019 08:04
03-10-2019 15:13
Harry CProfile picture★★☆☆☆
(157)
VernerHornung wrote:
tmiddles wrote:
Inevitable result of the law: "Those who can, do. Those who can't, criticize."

I'm in fact done with energy and Stefan-Boltzmann, satisfied what whatever might be wrong with the greenhouse doesn't involve these. Time to move on and find out what it is.

tmiddles wrote:
Earth is at 1013.25 millibars and Mars just 6.0 millibars pressure at ground level. That's just 0.6%. While CO2 is 0.04%, so Mars has 15 times the CO2. BUT the water vapor is 0-4% so that is overwhelming all told right? Really Mars makes a very solid case for Huffman's point, maybe more than Venus does.

The solar flux at 1 AU (Earth orbit) is 1360 W/m^2, so at 1.52 AU Mars will be getting 1360/1.52^2, or 589 Watts per square meter, just above the atmosphere and where the sun is directly overhead. Averaging over a sphere gives one-fourth this value, or 147 Watts, versus the 340 Watts the IPCC reports for each square meter on Earth. We can easily compute the blackbody temps:

T = (R/σ)^(1/4), with σ = 5.67 x 10^(-8) W / m^2 K^4

Earth: (340/5.67 x 10^(-8))^1/4 = 278K
Mars: (147/5.67 x 10^(-8)))^1/4 = 226K

So we have our first deviation from the blackbody situation: the albedo of each planet. On Earth, 30% of the R is reflected away without heating the surface, on Mars, 25%. So we redo the calculation with an adjustment for this:

Earth: (0.70 x 340/5.67 x 10^(-8))^1/4 = 255K
Mars: (0.75 x 147/5.67 x 10^(-8)))^1/4 = 210K

These match the "predicted temperatures" in the planet chart within a margin of rounding error. Earlier on the forum, I figured the greenhouse gases per square meter by mass, but I need to refine the formula a bit to account for CO2 being heavier than air and H2O lighter:

mass(CO2) = partial pressure (CO2) x molar mass (CO2) divided by molar mass (air ) divided by surface acceleration of gravity

Earth pressure 101.3 kPa, 0.04% CO2, 1.4% H2O
partial pressure of CO2 is 0.0004 x 101.3 kPa = 41 Pa
molar mass of CO2 = 44
molar mass of air = 29
gravity = 9.8 m/s^2
so mass (CO2) = (41 x 44) /(29 x 9.8) = 6.3 kg of CO2 per square meter

partial pressure of H2O is 0.014 x 101.3 kPa = 1420 Pa
molar mass of H2O = 18
so mass (H2O) = (1420 x 18) / (29 x 9.8) = 90 kg of H2O per square meter

Mars pressure 640 Pa, 95% CO2, 0.03% H2O
partial pressure of CO2 is 0.95 x 640 Pa = 608 Pa
molar mass of Martian air = 43.5
gravity = 3.7 m/s^2
so mass (CO2) = (608 x 44)/(43.5 x 3.7) = 166 kg of CO2 per square meter.

partial pressure of H2O is 0.0003 x 640 Pa = 0.19 Pa
so mass (H2O) = (0.19 x 18) / (43.5 x 3.7) = 0.021 kg of H2O per square meter



Mars has more greenhouse gases total overhead, per unit of surface area, than Earth does! These computations are rough because gravity decreases with height and other little things. Convection is also stronger, in relative terms, on Mars than on Earth because the air is very dry and nearly cloudless. (The clouds you see in the diagram are thin and there aren't many visible in rover photos.) the tropopause on Mars is over 30 km above ground, though the thinness of the air means convection doesn't carry much heat up from the surface.

But remember the pink sky of Viking? Mars has a load of junk in the air—no rain washing it out, so the Huffman-style direct absorption of sunlight by particles in the air will be important on Mars as well, if less so than on Venus & Earth. Global dust storms are known to drop surface temps by blocking sun; the Opportunity rover survived and took thermometer measurements during one. They heat the atmosphere aloft, however, as measured by the orbiters.

I dunno. I doubt direct heating of the air by sunlight produces the greenhouse effect itself, but it tends to decrease the temperature difference between air & ground, reducing convection. (A hot air balloon rises faster on a cold day.) Reduced convection, in turn, means a warmer surface because less heat from the ground is carried upward. The heating of air by sun warms the surface in an indirect way, and should do so without any greenhouse gases present.

That's my take on it, which is probably leaky as a sieve. Weather & climate is a complicated mess. Nobody in climate change has ever told us why the Martian "greenhouse" is only 6˚C with more gases than we have, and Earth's is 33˚. Reason could be the weaker sunlight there, or the optical thickness parameter in spectrography. (The Earth's CO2, despite its tiny concentration, is enough to block any 15-μm ray from reaching space from the ground, and such rays are also blocked on Mars.)

Seems greenhouse absorption/re-emission spectra, direct heating of air & clouds, vertical convection and horizontal winds all must be analyzed to compute a surface temp. It's beyond my ability.


That's an impressive array of formulas. My problem with it all is that you dismissed the science and are using anecdotal correlations to make a case for something you admit that you don't understand. What's the point? To obfuscate or convince yourself?


You learn something new every day if you are lucky!
03-10-2019 15:55
IBdaMannProfile picture★★★★★
(14404)
VernerHornung wrote:
IBdaMann wrote:
Then you are shifting semantic goalposts and are violating Stefan-Boltzman.

No moving goalposts here.

Yes there are, and it's obvious, ... they just aren't moving so fast that I don't notice. I quite clearly pointed out your specific goalpost-shifting.

Just as all warmizombies are want to do, you transition to violating Stefan-Boltzmann the moment someone points out that you are violating thermodynamics and you claim "See, I'm not violating thermodynamics!" ... at which point it is mentioned that your increase in temperature with a decrease in radiance violates Stefan-Boltzmann, ... and you immediately pivot back to violating thermodynamics while stating "See, I'm not violating Stefan-Boltzmann!" ... at which point it is mentioned that you have a sequence of energy form changes that somehow increases temperature via the creation of energy, which is a violation of thermodynamics, and you start all over by transitioning to violating Stefan-Boltzmann and claim "See, I'm not violating thermodynamics!" ... repeat ....

VernerHornung wrote: I've stated over and over again that the energy comes from the sun.

Just for laughs, in your mind, how many times do you have to tell yourself that the sun being the energy source qualifies you to violate the 1st law of thermodynamics? The earth simply does not have the power to either create or destroy energy. The amount of energy absorbed must equal the amount of energy radiated away. All of it. The moment you start playing your stupid games of dividing the earth in order to make the sum total emitted not equal the sum total absorbed, you are violating thermodynamics. The moment that you pivot and start claiming that reduced radiance equals increased temperature, you are violating Stefan-Boltzmann.

I'm guessing that tmiddles and yourself are the only ones you are fooling. You might be fooling spot as well, if he's still watching.

James__, is VernerHonung fooling you? I'm just asking.

VernerHornung wrote: And no violation of Stefan-Boltzmann because the Earth isn't a single radiating surface.

You repeatedly violate Stefan-Boltzmann by insisting that an increase in the earth's average global temperature is accompanied by a decrease in earth's total radiance.

Total, utter and complete violation of the Stefan-Boltzmann, time and time again. The amazing part is that it is the same violation you are committing every time. One would think that you would learn at some point. On that you certainly have everybody fooled.

VernerHornung wrote: It is a composite system with parts that radiate separately according to their own temperatures and emissivity characteristics.

Yes, it's comprised of matter ... made of a bajillion, bajillion molecules.

Yeah, so?

VernerHornung wrote: There may well be other problems with the atmospheric greenhouse theory,

Nope, at least not with your theory. You can't get beyond the two problems that I have detailed above through which you cycle. Either one alone garners a summary dismissal for your arguments.

.


I don't think i can [define it]. I just kind of get a feel for the phrase. - keepit

A Spaghetti strainer with the faucet running, retains water- tmiddles

Clouds don't trap heat. Clouds block cold. - Spongy Iris

Printing dollars to pay debt doesn't increase the number of dollars. - keepit

If Venus were a black body it would have a much much lower temperature than what we found there.- tmiddles

Ah the "Valid Data" myth of ITN/IBD. - tmiddles

Ceist - I couldn't agree with you more. But when money and religion are involved, and there are people who value them above all else, then the lies begin. - trafn

You are completely misunderstanding their use of the word "accumulation"! - Climate Scientist.

The Stefan-Boltzman equation doesn't come up with the correct temperature if greenhouse gases are not considered - Hank

:*sigh* Not the "raw data" crap. - Leafsdude

IB STILL hasn't explained what Planck's Law means. Just more hand waving that it applies to everything and more asserting that the greenhouse effect 'violates' it.- Ceist
03-10-2019 19:44
VernerHornungProfile picture★☆☆☆☆
(133)
Harry C wrote:
My problem with it all is that you dismissed the science and are using anecdotal correlations to make a case...

Hardly "anecdotal" at all. 25 successful missions to Mars have been flown which include long-term orbiters, three landers and four rovers. Mars Global Surveyor in particular observed Martian weather and created a map of global atmospheric circulation over 3 Martian years. You can peruse an archived website of this spacecraft's Thermal Emission Spectrometer instrument at

Arizona State U.
http://tes.asu.edu/discoveries/index.html

Geology discoveries, too, such as hematite & carbonate rocks. The latter would have "locked away" much of Mars' early CO2 as on Earth; the rock, basically limestone, forms only in water. Venus has had several orbiters and eight Soviet Venera landers. Data for Mercury are uncertain as only the Mariner 10 flybys from 1974-75 are available, though it seems to resemble our moon, if we could move the moon to Mercury's distance from the sun.

Harry C wrote:
What's the point? To obfuscate or convince yourself?

I'm not trying to "convince" myself of anything. I'm still agnostic regarding details of what causes climate change on a planet, of a dim view toward the environmental scare tactics swirling 'round industrial CO2 on Earth. The greenhouse effect is one such cause, but there are others, many of them natural, and the way energy is distributed between surface & air is complex—perhaps too complex to allow predictions of future surface temperatures to within 1˚C as IPCC wishes.

Harry C wrote:
...impressive array of formulas...

Only two formulas used, the rest is arithmetic. The idea was to verify the Earth & Mars temps shown on the American Chemical Society's toolkit pages here:



ACS
https://www.acs.org/content/acs/en/climatescience/energybalance/predictedplanetarytemperatures.html
https://www.acs.org/content/acs/en/climatescience/energybalance/predictedplanetarytemperatures.html

If you've had high school chemistry, you'll recall the ideal gas law which tells us that the partial pressure of a gas depends on the number of molecules in a sample when volume & temperature are held constant. I chose to convert this to mass per sq. meter. Pressure is just the weight of the gas resting above a unit area. For air at sea level, this is 14.7 lbs/in^2 or 101300 Pascals in metric units. Since CO2 is 0.04% of air by volume, its molecules contribute a skimpy 41 Pascals to the pressure.

Then you have the atomic weights—find them on any periodic table of the elements. For C, about 12, for N, about 14, for O, about 16 and for Ar, about 40. Now you can find the molar masses:

N2, 14 +14 = 28 (two atoms per molecule)
O2, 16 + 16 = 32
Ar, 40 (one atom per molecule)
CO2, 12 + 16 + 16 = 44 (three atoms)
air, 0.78 x 28 + 0.21 x 32 + 0.01 x 40 = 29

Air is the mixture of gases, percent of each gas times molecular weight, summed up. The percentages add up to 100% or 1.00, so air molecules weigh 29 atomic units on average. Since CO2 is heavier than air, we need the ratio 44/29 = 1.52 along with the partial pressure and acceleration of g, 9.8 m/s^2, at the surface.

Reason for finding the mass is that 1 kg of CO2 resting over a square yard of fabric laid out on the ground will absorb the same amount of infrared at a given wavelength no matter its volume and temperature. The column of CO2 can be 50 miles tall or just a foot, and it can be at Earthlike or Marslike temps. It's also easier to work with kilograms than with huge numbers of tiny molecules. Pressure of surrounding gases such as N2 and O2 will alter the absorption by broadening CO2's spectral lines.

Harry C wrote:
...something you admit that you don't understand.

Do you understand it? I hope to understand it better, and one thing to learn is why the greenhouse effect on Mars is so much weaker than Earth's when Mars actually has more gases than we do. The greenhouse theory is required to answer this question if we're to accept it at face value. Climate change literature doesn't go into this key issue.

The first steps were to find the blackbody temperatures and to find the masses of CO2 and H2O per square meter involved—documenting how I arrived at those results so the detractors on the forum can't claim I'm simply making up the numbers. Another forum user, Spot, pointed out the mass and computed it as a percentage figure:

New and 1st thoughts and questions
https://www.climate-debate.com/forum/new-and-1st-thoughts-and-questions-d6-e2798.php#post_44200

I'll take up the spectrum business next time.
~


IBdaMann wrote:
Just as all warmizombies are want to do, you transition to violating Stefan-Boltzmann...

You're raising nothing new and haven't shown where the violation occurs. I'll get back to you when I have time, but I won't discuss Stefan-Boltzmann until you explain the correct method for applying it to a body enclosed by a partially transparent shell.


Never try to solve an NP-complete problem on your own with pencil & paper.
Edited on 03-10-2019 19:49
03-10-2019 22:03
IBdaMannProfile picture★★★★★
(14404)
VernerHornung wrote:
Harry C wrote:
What's the point? To obfuscate or convince yourself?

I'm not trying to "convince" myself of anything. I'm still agnostic regarding details of what causes climate change on a planet,

... but you are a religious fundamentalist about that completely undefined and unfalsifiable concept of "climate change."


VernerHornung wrote: The greenhouse effect is one such cause,

Give me the "greenhouse effect" science. Just lay it out right here. Explain how the addition of a gas, not any additional energy, increases a body's average temperature.

.


I don't think i can [define it]. I just kind of get a feel for the phrase. - keepit

A Spaghetti strainer with the faucet running, retains water- tmiddles

Clouds don't trap heat. Clouds block cold. - Spongy Iris

Printing dollars to pay debt doesn't increase the number of dollars. - keepit

If Venus were a black body it would have a much much lower temperature than what we found there.- tmiddles

Ah the "Valid Data" myth of ITN/IBD. - tmiddles

Ceist - I couldn't agree with you more. But when money and religion are involved, and there are people who value them above all else, then the lies begin. - trafn

You are completely misunderstanding their use of the word "accumulation"! - Climate Scientist.

The Stefan-Boltzman equation doesn't come up with the correct temperature if greenhouse gases are not considered - Hank

:*sigh* Not the "raw data" crap. - Leafsdude

IB STILL hasn't explained what Planck's Law means. Just more hand waving that it applies to everything and more asserting that the greenhouse effect 'violates' it.- Ceist
04-10-2019 00:29
Into the NightProfile picture★★★★★
(21596)
VernerHornung wrote:
IBdaMann wrote:
Then you are shifting semantic goalposts and are violating Stefan-Boltzman.

No moving goalposts here.

Lie. You are moving goalposts from the Sun to magick gases in the atmosphere and back again.
VernerHornung wrote:
I've stated over and over again that the energy comes from the sun.

You've also stated over and over that energy comes from magick gases in the atmosphere.
VernerHornung wrote:
And no violation of Stefan-Boltzmann because the Earth isn't a single radiating surface.

Yes it is.
VernerHornung wrote:
It is a composite system with parts that radiate separately according to their own temperatures and emissivity characteristics.

According to Kirchoff's law, all energy nodes are effectively combined into one single energy node, which is the sum of all of its parts. You can't move the goalposts around by separating out parts and then considering them as the whole and back again.
VernerHornung wrote:
This isn't something I invented on my own;

Yes it is.
VernerHornung wrote:
it's been developed by chemists and physicists

Argument from randU fallacy. It took only a dozen or so men to develop the laws of thermodynamics and the Stefan-Boltzmann law.
VernerHornung wrote:
since Svante Arennhius (1896),

Please don't misquote this poor man again. I tire of it.
VernerHornung wrote:
becoming a matter of interest to planetary astronomers long before the political controversies over carbon arose.

Big deal. Science isn't a scientist, an astronomer, or any group of them. It is not people at all. It is just the set of falsifiable theories. You cannot use 'consensus' to deny the 1st and 2nd laws of thermodynamics or the Stefan-Boltzmann law.
VernerHornung wrote:
Meaning it wasn't invented to support the climate change agenda, either.

Define 'climate change'.
VernerHornung wrote:
There may well be other problems with the atmospheric greenhouse theory, but ex nihilo energy creation and Stefan-Boltzmann are not among them.

It is the problem. You can't simply ignore them. Not even an ostrich will stick his head in the sand, but you do, hoping these theories will simply go away.


The Parrot Killer

Debunked in my sig. - tmiddles

Google keeps track of paranoid talk and i'm not on their list. I've been evaluated and certified. - keepit

nuclear powered ships do not require nuclear fuel. - Swan

While it is true that fossils do not burn it is also true that fossil fuels burn very well - Swan
04-10-2019 00:31
Into the NightProfile picture★★★★★
(21596)
tmiddles wrote:
Into the Night wrote:
Absorption of visible light doesn't convert to thermal energy.
So what does it convert to?

VernerHornung wrote:...Ever notice how the Man and the Parrot Killer keep trashing my art?...
Inevitable result of the law: "Those who can, do. Those who can't, criticize."

VernerHornung wrote:
Ja. I think Mars is a better test case, though. The surface is visible from space, with full daylight sun. It has seasons like Earth's and a similar rotation period. It has a similar amount of total greenhouse gases because its air, though thin, is all CO2. Yet it has less of a greenhouse effect than Earth does. That's partly the weaker sun at its orbit, and because it has no significant water vapor.
So we have Mercury, with no atmosphere, giving us 4 degrees more than what we'd expect for the distance from the Sun. Venus is crazy town. Earth is 30 degrees above and Mars is just 6 degrees above.
Really with Mercury coming in at 4 degrees above and what has to be a healthy margin of error Mars isn't really doing ANYTHING in observed vs. calculated.

Earth is at 1013.25 millibars and Mars just 6.0 millibars pressure at ground level. That's just 0.6%. While CO2 is 0.04%, so Mars has 15 times the CO2. BUT the water vapor is 0-4% so that is overwhelming all told right? Really Mars makes a very solid case for Huffman's point, maybe more than Venus does.

VernerHornung wrote:
applying it to planets with dynamic atmospheres is difficult and that's what makes me skeptical of getting precise values for surface temperature rise as you add CO2 to Earth's atmosphere in a computer model.
Yes and any theory being modeled should also get the other planets right as well.

VernerHornung wrote:
Into the Night wrote:
You can't trap or slow heat.

I'll keep that in mind every time I have to put my coat on this winter.~
ITN likes to be deceptive with vocabulary games. I think he'd say you can "reduce heat". But it's all a feverish attempt to claim thermal energy cannot build up anywhere like we warmazombies so passionately want it to.

Remember that in IBD/ITN land that coat won't help you. You're a goner in even a 70F room!



RQAA. RDCF. YALIF


The Parrot Killer

Debunked in my sig. - tmiddles

Google keeps track of paranoid talk and i'm not on their list. I've been evaluated and certified. - keepit

nuclear powered ships do not require nuclear fuel. - Swan

While it is true that fossils do not burn it is also true that fossil fuels burn very well - Swan
04-10-2019 00:32
Into the NightProfile picture★★★★★
(21596)
VernerHornung wrote:
tmiddles wrote:
Inevitable result of the law: "Those who can, do. Those who can't, criticize."

I'm in fact done with energy and Stefan-Boltzmann, satisfied what whatever might be wrong with the greenhouse doesn't involve these. Time to move on and find out what it is.


Of course you are. It gets in your way, so you just deny it.


The Parrot Killer

Debunked in my sig. - tmiddles

Google keeps track of paranoid talk and i'm not on their list. I've been evaluated and certified. - keepit

nuclear powered ships do not require nuclear fuel. - Swan

While it is true that fossils do not burn it is also true that fossil fuels burn very well - Swan
04-10-2019 00:35
Into the NightProfile picture★★★★★
(21596)
Harry C wrote:
VernerHornung wrote:
tmiddles wrote:
Inevitable result of the law: "Those who can, do. Those who can't, criticize."

I'm in fact done with energy and Stefan-Boltzmann, satisfied what whatever might be wrong with the greenhouse doesn't involve these. Time to move on and find out what it is.

tmiddles wrote:
Earth is at 1013.25 millibars and Mars just 6.0 millibars pressure at ground level. That's just 0.6%. While CO2 is 0.04%, so Mars has 15 times the CO2. BUT the water vapor is 0-4% so that is overwhelming all told right? Really Mars makes a very solid case for Huffman's point, maybe more than Venus does.

The solar flux at 1 AU (Earth orbit) is 1360 W/m^2, so at 1.52 AU Mars will be getting 1360/1.52^2, or 589 Watts per square meter, just above the atmosphere and where the sun is directly overhead. Averaging over a sphere gives one-fourth this value, or 147 Watts, versus the 340 Watts the IPCC reports for each square meter on Earth. We can easily compute the blackbody temps:

T = (R/σ)^(1/4), with σ = 5.67 x 10^(-8) W / m^2 K^4

Earth: (340/5.67 x 10^(-8))^1/4 = 278K
Mars: (147/5.67 x 10^(-8)))^1/4 = 226K

So we have our first deviation from the blackbody situation: the albedo of each planet. On Earth, 30% of the R is reflected away without heating the surface, on Mars, 25%. So we redo the calculation with an adjustment for this:

Earth: (0.70 x 340/5.67 x 10^(-8))^1/4 = 255K
Mars: (0.75 x 147/5.67 x 10^(-8)))^1/4 = 210K

These match the "predicted temperatures" in the planet chart within a margin of rounding error. Earlier on the forum, I figured the greenhouse gases per square meter by mass, but I need to refine the formula a bit to account for CO2 being heavier than air and H2O lighter:

mass(CO2) = partial pressure (CO2) x molar mass (CO2) divided by molar mass (air ) divided by surface acceleration of gravity

Earth pressure 101.3 kPa, 0.04% CO2, 1.4% H2O
partial pressure of CO2 is 0.0004 x 101.3 kPa = 41 Pa
molar mass of CO2 = 44
molar mass of air = 29
gravity = 9.8 m/s^2
so mass (CO2) = (41 x 44) /(29 x 9.8) = 6.3 kg of CO2 per square meter

partial pressure of H2O is 0.014 x 101.3 kPa = 1420 Pa
molar mass of H2O = 18
so mass (H2O) = (1420 x 18) / (29 x 9.8) = 90 kg of H2O per square meter

Mars pressure 640 Pa, 95% CO2, 0.03% H2O
partial pressure of CO2 is 0.95 x 640 Pa = 608 Pa
molar mass of Martian air = 43.5
gravity = 3.7 m/s^2
so mass (CO2) = (608 x 44)/(43.5 x 3.7) = 166 kg of CO2 per square meter.

partial pressure of H2O is 0.0003 x 640 Pa = 0.19 Pa
so mass (H2O) = (0.19 x 18) / (43.5 x 3.7) = 0.021 kg of H2O per square meter



Mars has more greenhouse gases total overhead, per unit of surface area, than Earth does! These computations are rough because gravity decreases with height and other little things. Convection is also stronger, in relative terms, on Mars than on Earth because the air is very dry and nearly cloudless. (The clouds you see in the diagram are thin and there aren't many visible in rover photos.) the tropopause on Mars is over 30 km above ground, though the thinness of the air means convection doesn't carry much heat up from the surface.

But remember the pink sky of Viking? Mars has a load of junk in the air—no rain washing it out, so the Huffman-style direct absorption of sunlight by particles in the air will be important on Mars as well, if less so than on Venus & Earth. Global dust storms are known to drop surface temps by blocking sun; the Opportunity rover survived and took thermometer measurements during one. They heat the atmosphere aloft, however, as measured by the orbiters.

I dunno. I doubt direct heating of the air by sunlight produces the greenhouse effect itself, but it tends to decrease the temperature difference between air & ground, reducing convection. (A hot air balloon rises faster on a cold day.) Reduced convection, in turn, means a warmer surface because less heat from the ground is carried upward. The heating of air by sun warms the surface in an indirect way, and should do so without any greenhouse gases present.

That's my take on it, which is probably leaky as a sieve. Weather & climate is a complicated mess. Nobody in climate change has ever told us why the Martian "greenhouse" is only 6˚C with more gases than we have, and Earth's is 33˚. Reason could be the weaker sunlight there, or the optical thickness parameter in spectrography. (The Earth's CO2, despite its tiny concentration, is enough to block any 15-μm ray from reaching space from the ground, and such rays are also blocked on Mars.)

Seems greenhouse absorption/re-emission spectra, direct heating of air & clouds, vertical convection and horizontal winds all must be analyzed to compute a surface temp. It's beyond my ability.


That's an impressive array of formulas. My problem with it all is that you dismissed the science and are using anecdotal correlations to make a case for something you admit that you don't understand. What's the point? To obfuscate or convince yourself?


To obfuscate. This sort of tactic is typical of some of the Faithful. It is designed to make them look 'scientific'. In this case, it's to help cover up the fact that he denies the Stefan-Boltzmann law.


The Parrot Killer

Debunked in my sig. - tmiddles

Google keeps track of paranoid talk and i'm not on their list. I've been evaluated and certified. - keepit

nuclear powered ships do not require nuclear fuel. - Swan

While it is true that fossils do not burn it is also true that fossil fuels burn very well - Swan
04-10-2019 00:55
Into the NightProfile picture★★★★★
(21596)
VernerHornung wrote:
Harry C wrote:
My problem with it all is that you dismissed the science and are using anecdotal correlations to make a case...

Hardly "anecdotal" at all.
...deleted lengthy anecdotes...

Lie. You wandered into lengthy anecdotes AGAIN in your response!
VernerHornung wrote:
Harry C wrote:
What's the point? To obfuscate or convince yourself?

I'm not trying to "convince" myself of anything. I'm still agnostic regarding details of what causes climate change on a planet, of a dim view toward the environmental scare tactics swirling 'round industrial CO2 on Earth. The greenhouse effect is one such cause,

Claiming that are agnostic and then assuming Jesus Christ exists is not being agnostic. Claiming to be agnostic and then assuming 'climate change' is not being agnostic.
VernerHornung wrote:
...deleted remaining religious 'proofs' and assumptions...

Being a fundamentalist believer in the Church of Global Warming isn't being agnostic either.
Harry C wrote:
...impressive array of formulas...

Only two formulas used, the rest is arithmetic. The idea was to verify the Earth & Mars temps shown on the American Chemical Society's toolkit pages here:
...deleted lengthy array of unrelated equations and formulas...
Pressure of surrounding gases such as N2 and O2 will alter the absorption by broadening CO2's spectral lines.
[/quote]
Another lie. You wandered off into yet another lengthy array of equations and formulas.

Pressure does NOT change absorption spectra at all.
VernerHornung wrote:
Harry C wrote:
...something you admit that you don't understand.

Do you understand it? I hope to understand it better, and one thing to learn is why the greenhouse effect on Mars is so much weaker than Earth's when Mars actually has more gases than we do. The greenhouse theory is required to answer this question if we're to accept it at face value. Climate change literature doesn't go into this key issue.

Contextomy fallacy. It is YOU that claiming to 'understand' it. His 'understanding' of it is not a factor. YOUR claim does not depend on his status.

No gas or vapor is capable of warming any planet using that planet's emitted IR from the surface.
* You can't create energy out of nothing.
* You cannot trap light.
* You cannot reduce the radiance and increase the temperature at the same time.

There is no 'greenhouse' theory. The 'greenhouse effect' is based on the phrase 'global warming', which remains undefined. No theory of any kind may be based on a void argument fallacy (internal consistency check).
VernerHornung wrote:
The first steps were to find the blackbody temperatures and to find the masses of CO2 and H2O per square meter involved—documenting how I arrived at those results so the detractors on the forum can't claim I'm simply making up the numbers. Another forum user, Spot, pointed out the mass and computed it as a percentage figure:

The emissivity of Mars is unknown. The emissivity of Earth is unknown.
VernerHornung wrote:
New and 1st thoughts and questions
https://www.climate-debate.com/forum/new-and-1st-thoughts-and-questions-d6-e2798.php#post_44200

I'll take up the spectrum business next time.
~


There is no frequency term in the Stefan-Boltzmann law.
VernerHornung wrote:
IBdaMann wrote:
Just as all warmizombies are want to do, you transition to violating Stefan-Boltzmann...

You're raising nothing new and haven't shown where the violation occurs.

I just did. IBdaMann has also shown you. RQAA.
VernerHornung wrote:
I'll get back to you when I have time, but I won't discuss Stefan-Boltzmann until you explain the correct method for applying it to a body enclosed by a partially transparent shell.

Planets don't have partially transparent shells. It is irrelevant anyway. The Stefan-Boltzmann law applies the same way, shell and all.


The Parrot Killer

Debunked in my sig. - tmiddles

Google keeps track of paranoid talk and i'm not on their list. I've been evaluated and certified. - keepit

nuclear powered ships do not require nuclear fuel. - Swan

While it is true that fossils do not burn it is also true that fossil fuels burn very well - Swan
04-10-2019 13:59
VernerHornungProfile picture★☆☆☆☆
(133)
Into the Night wrote:
According to Kirchoff's law, all energy nodes are effectively combined into one single energy node, which is the sum of all of its parts. You can't move the goalposts around by separating out parts and then considering them as the whole and back again.

Kirchoff's law for radiation was derived from a radiative equilibrium between two emitting and absorbing infinite parallel plates of different materials facing each other. There were already two bodies at stake. His conclusion said that for rays of the same wavelength at the same temperature the ratio of the emissive power to the absorptivity is the same for all bodies. But he did not say that all bodies have the same emissive power at a particular wavelength. If the emissivity happens to be low at that wavelength, then the absorptivity will also be low.

Furthermore, you can add blackbody spectra to one another by superposition of waves to get a resultant temperature and spectrum. This implies (by division algorithm in arithmetic) that you can divide them wavelength by wavelength as well, if you want temperature independence, and the THEMIS instrument on Mars Odyssey did so to obtain characteristic spectra for minerals on Mars, including those of carbonate rocks that form in water.

Into the Night wrote:
Planets don't have partially transparent shells.

Yes, they do. They're called atmospheres.


Never try to solve an NP-complete problem on your own with pencil & paper.
Edited on 04-10-2019 14:49
04-10-2019 14:23
VernerHornungProfile picture★☆☆☆☆
(133)
Into the Night wrote:
Pressure does NOT change absorption spectra at all.

Yes, it does:

"The spectra there are computed from the HITRAN database with lines broadened by collision with air at a pressure of 5 x 10^4 Pa..."

Infrared Radiation & Planetary Temperature
Raymond Pierrehumbert
Department of Geophysical Sciences
University of Chicago
https://geosci.uchicago.edu/~rtp1/papers/PhysTodayRT2011.pdf
(see p. 3 in pdf file)

You've made that claim twice and I've replied citing this source twice. But it won't do any good; you're gonna repeat your absurdities like a broken record until they become true—a tactic the radicals use to get the public to accept their claims about society. Interesting how little the far right and the far left differ from one another, when politicizing an issue, even if it's one which should be a matter of science.


Never try to solve an NP-complete problem on your own with pencil & paper.
04-10-2019 20:36
Into the NightProfile picture★★★★★
(21596)
VernerHornung wrote:
Into the Night wrote:
According to Kirchoff's law, all energy nodes are effectively combined into one single energy node, which is the sum of all of its parts. You can't move the goalposts around by separating out parts and then considering them as the whole and back again.

Kirchoff's law for radiation was derived from a radiative equilibrium between two emitting and absorbing infinite parallel plates of different materials facing each other. There were already two bodies at stake. His conclusion said that for rays of the same wavelength at the same temperature the ratio of the emissive power to the absorptivity is the same for all bodies. But he did not say that all bodies have the same emissive power at a particular wavelength. If the emissivity happens to be low at that wavelength, then the absorptivity will also be low.

Irrelevant. You are ignoring Kirchoff's law of summation of energy nodes. You are also changing the boundaries of a closed system at your convenience. Not allowed in thermodynamics. The system must be consistent and closed. You are moving the goalposts.

You may consider the surface, OR you may consider the atmosphere, but you cannot consider them in total unless you are ALWAYS considering them in total. You can't compare a piece to the whole. You are moving the goalposts. False equivalence fallacy. Goalpost fallacy.
VernerHornung wrote:
Furthermore, you can add blackbody spectra to one another by superposition of waves to get a resultant temperature and spectrum.

You are now attempting to misuse Wien's law. This doesn't work to determine temperature accurately. You don't know how much light is reflected from a body, and you do not know how much light is generated from the body using other means than black body radiance. We can use it for stars because we only care about an estimate +- some thousands of degrees. We can use it for metalwork +- 50degF or so IF we carefully control the light the hot metal is being viewed in.

VernerHornung wrote:
This implies (by division algorithm in arithmetic) that you can divide them wavelength by wavelength as well, if you want temperature independence, and the THEMIS instrument on Mars Odyssey did so to obtain characteristic spectra for minerals on Mars, including those of carbonate rocks that form in water.

Does not give you the temperature.
VernerHornung wrote:
Into the Night wrote:
Planets don't have partially transparent shells.

Yes, they do. They're called atmospheres.

Okay., you are considering an atmosphere a 'shell'. Why not call it an atmosphere? Obviously, you are trying to separate the same thing into two different things at your convenience.

It's irrelevant anyway. As I said before, it makes no difference.


The Parrot Killer

Debunked in my sig. - tmiddles

Google keeps track of paranoid talk and i'm not on their list. I've been evaluated and certified. - keepit

nuclear powered ships do not require nuclear fuel. - Swan

While it is true that fossils do not burn it is also true that fossil fuels burn very well - Swan
Edited on 04-10-2019 20:39
04-10-2019 20:52
Into the NightProfile picture★★★★★
(21596)
VernerHornung wrote:
Into the Night wrote:
Pressure does NOT change absorption spectra at all.

Yes, it does:

No, it doesn't.
VernerHornung wrote:
"The spectra there are computed from the HITRAN database with lines broadened by collision with air at a pressure of 5 x 10^4 Pa..."

Infrared Radiation & Planetary Temperature
Raymond Pierrehumbert
Department of Geophysical Sciences
University of Chicago
https://geosci.uchicago.edu/~rtp1/papers/PhysTodayRT2011.pdf
(see p. 3 in pdf file)

This distortion is caused by measuring something other than CO2 absorption. Contextomy fallacy.
VernerHornung wrote:
You've made that claim twice and I've replied citing this source twice.

And you've made the same contextomy fallacy twice.
VernerHornung wrote:
But it won't do any good; you're gonna repeat your absurdities like a broken record until they become true—a tactic the radicals use to get the public to accept their claims about society.

Inversion fallacy.
VernerHornung wrote:
Interesting how little the far right and the far left differ from one another, when politicizing an issue, even if it's one which should be a matter of science.

This paper isn't science. Neither is the use of instruments to take measurements. Science isn't data. It isn't a piece of paper with writing on it. It is a set of falsifiable theories. No falsifiable theory of any kind was presented by this paper. Just observations and preconclusions. Circular argument fallacy (fundamentalism). Use of observation as proof.

Observations are subject to the problems of phenomenology. This particular paper is an excellent example of just this sort of problem. Observations are not a proof. They are evidence only. Science uses no supporting evidence. You cannot use any evidence to prove any theory True.


The Parrot Killer

Debunked in my sig. - tmiddles

Google keeps track of paranoid talk and i'm not on their list. I've been evaluated and certified. - keepit

nuclear powered ships do not require nuclear fuel. - Swan

While it is true that fossils do not burn it is also true that fossil fuels burn very well - Swan
05-10-2019 06:24
VernerHornungProfile picture★☆☆☆☆
(133)
Into the Night wrote:
You are now attempting to misuse Wien's law. This doesn't work to determine temperature accurately.

Then how do you explain this?



According to Wien's law, a 320K blackbody has its peak emission at

λ = (2.898 x 10^-3 m∙K) / 320K = 9.05 x 10^-6 m

or about 9μm, if the peak is sought by units of wavelength, or

ν = 5.879 x 10^10 Hz/K x 320K = 1.88 x 10^13 Hz,

which corresponds to 15.9μm if you prefer units of frequency in locating the peak, as was done in this graph. It was obtained in 1970 by a Nimbus weather satellite as it passed 1000 km over the Niger valley on what looks like a hot day there. Either way you parametrize it, the 320K curve in this image, which I did not create, shows the peak as chopped off. (To re-parametrize to units of λ, just multiply all the numbers along the top by 0.568 and divide all the numbers along the bottom by 0.568. The Planck curve does not change shape whether graphed by wavelength or frequency.)

Of course the peak is still where Wien's law says it should be, were it not missing because of the big dip in the graph marked CO2. The bottom of this dip lies on the downslope of the 210K curve instead. What does this tell you? The radiation in the dip zone is coming to the satellite from a different place, high in the atmosphere, than the rays to its right, which come from near the surface and match the 320K curve.

Dividing the area under the squiggly line by the area under the 320K curve gives the emissivity ε of the composite system. As you've probably guessed, the value of ε obtained this way changes with location on Earth, season and weather, a complexity that means finding a value to use for Earth as whole, averaged over all seasons & places, is difficult. However, planets clothed with atmospheres emit as if their surfaces were cooler than they really are.

Color vision researchers do superposition of waves, by the way. A test subject views light from red and green lamps through prism binoculars that cause the lamps' images to merge on his retinas, and sees a yellow color.

Into the Night wrote:
Irrelevant. You are ignoring Kirchoff's law of summation of energy nodes. You are also changing the boundaries of a closed system at your convenience.

The Earth is not a closed system. And you can draw a boundary around a system wherever you please. There are no preferred systems, nor closed ones save the universe itself. Thermos-bottle type chambers with highly reflective walls approximate closure for a period of time in a lab when needed, and an experimenter chooses what to put inside the bottle.

The nodes are simply the points at which the waves have zero instantaneous amplitude. In the parallel plates situation at thermal equilibrium (or in a cavity radiator also), the two endpoint nodes of each wave much touch the walls lest a current be induced there, when we would then have net current flowing into or out from a point on the wall in violation of Kirchoff's laws for electric circuits. (Cavity radiators are made of metal, incidentally.)

That's beside the point. While it's convenient to use Planck curves to analyze radiation exchange, no real-world object is a simple blackbody, especially if it's an unionized gas. In solids, the atoms are strongly coupled through bonding, and in stars, ionization creates a conductive medium, the plasma, consisting of electrons, protons and positive ions. Stars therefore come the closest to blackbody behavior, with solids often acting as a "gray" body that emits at all frequencies but with less efficiency. Because molecules in gases are isolated except during collisions, they must treated with respect to their spectral properties and the scattering they cause. If curious, the radiative transfer equation is used for them:



This equation must be evaluated for every direction of travel at each point and results integrated for the body's total absorption and emission. Local, but not global, thermodynamic equilibrium is assumed, meaning the gas is isothermal within a neighborhood of the point in question. The absorption and emission are functions of the radiation's frequency ν and may not be equal as they were in simple blackbody situations, although both are proportional to the density. In gases, the density will depend on temperature and pressure. The variable s represents arc length along the path the radiation takes through the gas. (That's small s, not big S, which is the source function.)

For use in climate models, I believe the air is sliced into thin layers as the diagram below suggests, and calculations are run for each layer.



IBdaMann wrote:
The amount of energy absorbed must equal the amount of energy radiated away. All of it.

And it is. The sunlight at our orbit is only sufficient to heat a gray body with an emissivity constant of 0.70 to 255K, or 0˚F. Even a perfect blackbody would reach only 278K, or 41˚F. I computed both of these temps in my last post this thread. According to your ideas on the topic, Earth should therefore the kind of chilly place that suits your personality. Yet it isn't. Why?

I'll clue you in. Earth isn't a blackbody and some mechanism enables its surface to stay warmer than if it were naked like the moon's. Good thing. Radiative balance with the sun for the entire planet need hold only at the top of the atmosphere, because that's really where Earth ends and space begins.

IBdaMann wrote:
Explain how the addition of a gas, not any additional energy, increases a body's average temperature.

If you just add gas, no temperature hike. You need a star, and a gas transparent to the bulk of the star's light but opaque to a portion of the planet's return radiation.

IBdaMann wrote:
Give me the "greenhouse effect" science. Just lay it out right here.

The basic equation for the radiation is above; you will also need equations for the drops in pressure and temperature of air with altitude. I wish it could be rendered childishly easy for your manual's future entry on it.


Never try to solve an NP-complete problem on your own with pencil & paper.
05-10-2019 07:39
VernerHornungProfile picture★☆☆☆☆
(133)
Into the Night wrote:
Observations are subject to the problems of phenomenology. This particular paper is an excellent example of just this sort of problem. Observations are not a proof.


Infrared absorption by CO2 has been measured in the lab under controlled conditions. It's known that this gas absorbs strongly at wave numbers 670 and 2240. Very little of the sun's output lies below wave number 30000, and CO2 is in fact transparent in daylight. It is also known that visible light can heat solid objects—blue-green medical argon lasers are used to cauterize tiny blood vessels in diabetic retinopathy to stop fluid leakage that causes blindness. Neither of these phenomena remain solely empirical, that is, limited to observational results. The heating can be calculated, and lab results agree with the calculations within reasonable experimental error.


NIST infrared spectrum of CO2

Pierrehumbert's paper is the best non-mathematical explanation of the physics behind greenhouse effects I've found. The Cassini spacecraft observed the predicted warmth of Titan's surface, where N2 serves as the greenhouse gas because of its ability to absorb in far-IR and microwave bands at the frigid temps prevalent there.

We can't prove that climate change on Earth in recent decades isn't caused at least in part by something other than anthropogenic greenhouse gas emissions. Regional climate is affected by El Nino cycles and the North Atlantic Oscillation, for instance, which occur for poorly understood reasons. But that's a different story, one of the applicability of known physics & chemistry to complex systems.


Never try to solve an NP-complete problem on your own with pencil & paper.
05-10-2019 16:43
IBdaMannProfile picture★★★★★
(14404)
VernerHornung wrote: Infrared absorption by CO2 has been measured in the lab under controlled conditions. It's known that this gas absorbs strongly at wave numbers 670 and 2240.

I don't think you get the point.

I have not seen anyone dispute your assertion about the properties of CO2.

The question is "so what?"

What you need to explain is how the earth is able to somehow destroy or create energy (in violation of the 1st law of thermodynamics) by your obsession with focusing on atmospheric CO2 ...

*AND*

... how the earth can lower in radiance while increasing in temperature (in violation of Stefan-Boltzmann) by your obsession with focusing on atmospheric CO2).

Until you explain those, EVERYTHING ELSE is summarily dismissed.

VernerHornung wrote: Pierrehumbert's paper is the best non-mathematical explanation of the physics behind greenhouse effects I've found.

What does he say that answers the above? Summarize his physics behind "Greenhouse Effect."


VernerHornung wrote: The Cassini spacecraft observed the predicted warmth of Titan's surface,

Nope. It possibly observed a speculated "warmth" (I really wish you wouldn't get so technical).

Learn what "predict" means in physics.

[hint: calculated from a falsifiable model ... which requires a falsifiable model]


VernerHornung wrote: We can't prove that climate change on Earth in recent decades isn't caused at least in part by something other than anthropogenic greenhouse gas emissions.

In fact, you can't even unambiguously define what the fug you're talking about.

... but your single biggest problem is that this doesn't bother you. When warmizombies preach to you about the good news of Global Warming, you won't question a single premise. You simply obey whoever is doing your thinking for you.

VernerHornung wrote: Regional climate is affected by ...

Let's stay focused on global "climate." What is that EXACTLY?

.


I don't think i can [define it]. I just kind of get a feel for the phrase. - keepit

A Spaghetti strainer with the faucet running, retains water- tmiddles

Clouds don't trap heat. Clouds block cold. - Spongy Iris

Printing dollars to pay debt doesn't increase the number of dollars. - keepit

If Venus were a black body it would have a much much lower temperature than what we found there.- tmiddles

Ah the "Valid Data" myth of ITN/IBD. - tmiddles

Ceist - I couldn't agree with you more. But when money and religion are involved, and there are people who value them above all else, then the lies begin. - trafn

You are completely misunderstanding their use of the word "accumulation"! - Climate Scientist.

The Stefan-Boltzman equation doesn't come up with the correct temperature if greenhouse gases are not considered - Hank

:*sigh* Not the "raw data" crap. - Leafsdude

IB STILL hasn't explained what Planck's Law means. Just more hand waving that it applies to everything and more asserting that the greenhouse effect 'violates' it.- Ceist
06-10-2019 07:44
tmiddlesProfile picture★★★★★
(3979)
VernerHornung wrote: So we redo the calculation with an adjustment for this:

Earth: (0.70 x 340/5.67 x 10^(-8))^1/4 = 255K
Mars: (0.75 x 147/5.67 x 10^(-8)))^1/4 = 210K

These match the "predicted temperatures" in the planet chart within a margin of rounding error.
Just gotta say nicely done!

VernerHornung wrote:
Mars has more greenhouse gases total overhead, per unit of surface area, than Earth does!
This really seems to corroborate Huffman's theory that it's all about total mass. As well as Blair McDonald here saying that the other gasses are also producing a greenhouse effect: So the "non-greenhouse" gases are also greenhouse gases to some degree

VernerHornung wrote:
The heating of air by sun warms the surface in an indirect way, and should do so without any greenhouse gases present.
It seem pretty direct to me. If the air is hotter then the ground will be too right?

VernerHornung wrote:Nobody in climate change has ever told us why the Martian "greenhouse" is only 6˚C with more gases than we have, and Earth's is 33˚.
I wonder if anyone has modeled it?

I'm still frustrated that you can't do lab experiments to demonstrate this stuff. Forget about finding out why any planet is doing something. Just predict and model Plexiglas boxes full of precise gas mixes with black paper a the back. I'm guessing someone is doing this type of work.

Harry C wrote:...My problem with it all is that you dismissed the science and are using anecdotal correlations to make a case for something you admit that you don't understand. What's the point? To obfuscate or convince yourself?
What do you mean "dismissed the science"? Verner and I are wondering about something. Why is it that way? We don't know. That's OK. It's SCIENCE : D

This stuff isn't figured out yet as far as we know. I would love to know more! How about you?

IBdaMann wrote:...you are violating thermodynamics...
IBD, who learned his private thermodynamics from a guy on the street in the oral tradition of Physics he adheres too. No book, no research, just word o' mouth.
IBdaMann wrote:
tmiddles wrote:...How did you learn the Stefan-Boltzmann Law ...
Someone explained the science to me,...
In 5 years never a single calculation done.

IBdaMann wrote:
VernerHornung wrote: I've stated over and over again that the energy comes from the sun.
... violate the 1st law of thermodynamics? The earth simply does not have the power to either create or destroy energy.
You simply deny that the Earth is 30 degrees warmer than it should be. Oh no we don't know the temp of Earth or of anything they cry! We can't talk about it they cry! Lame lame lame.

No explanation for you guys at all. Just BS.


"Good tests kill flawed theories; we remain alive to guess again." - Karl Popper
ITN/IBD Fraud exposed:  The 2nd LTD add on claiming radiance from cooler bodies can't be absorbed Max Planck debunks, they can't explain:net-thermal-radiation-you-in-a-room-as-a-reference & Proof: no data is ever valid for them
06-10-2019 09:08
tmiddlesProfile picture★★★★★
(3979)
VernerHornung wrote:
Harry C wrote:
What's the point? To obfuscate or convince yourself?

I'm not trying to "convince" myself of anything. I'm still agnostic regarding details of what causes climate change on a planet, of a dim view toward the environmental scare tactics swirling 'round industrial CO2 on Earth.

I concur and have a similar position at this point. One trap extremists fall into, both the ITN/IBDs of the world and the crying sky is falling environmental doomsdayers, is that they over reach with their claims and end up in an argument where they are dead wrong.
Something can be TRUE and yet be INSIGNIFICANT. If you claim it doesn't exist, and it does, you are doing a great job of creating the illusion that it's VERY SIGNIFICANT since you seem so afraid of it. So it goes with made up science arguments from buffoons like ITN/IBD. Real skeptical scientists appear behind a cloud of that BS.
A perfect example of confusing "it exists" with "it is important" is with radiation. Radiation is real! When Fukushima happened you bet some of that radiation got into the coast waters near the plant. Some very stupid people said don't go in the pacific ocean on the US west coast, don't eat fish. This is effectively holding up a MORON sign and discrediting anything that ever comes out of your mouth on the subject again. There is NO upside for anyone when this kind of human stupidity flourishes.

VernerHornung wrote:
Harry C wrote:
...something you admit that you don't understand.

Do you understand it? I hope to understand it better, and one thing to learn is why the greenhouse effect on Mars is so much weaker than Earth's when Mars actually has more gases than we do.
Yes having the other planets seems like our best resource in being able to figure out how it all works. They are real, full, equally complex models we can study. And for what it's worth Verner I'm understanding the basic science a lot better thanks to your work.

Also Harry this was actually "Step one" in the whole science of temperature and atmosphere. 1827 Fourier indicated the similarity between what happens under the glass of a greenhouse and how heat is absorbed in the atmosphere. This led to the term "greenhouse effect.".

Fourier noticed that the Earth was warmer that is should be.

He didn't have the direct measurements from Venus and Mars but he was still able to realize there was an odd extra 30 degrees to our mean temperature at ground level.

VernerHornung wrote:
IBdaMann wrote:....violating Stefan-Boltzmann...
...you explain the correct method for applying it to a body enclosed by a partially transparent shell.
Don't hold your breath Verner. IBD has never explained how to do anything in 5 years.

IBdaMann wrote:Explain how the addition of a gas, not any additional energy, increases a body's average temperature.
What was your theory on Venus? Oh right, nothing, nothing at all.
IBdaMann wrote:
Venus' atmosphere is irrelevant.


Into the Night wrote:
VernerHornung wrote:
It is a composite system with parts...
According to Kirchoff's law, ...You can't move the goalposts around by separating out parts and then considering them as the whole and back again.
This is completely unfounded. You can consider everything in the universe to be one "part" and you can consider a single atom to be one "part". Stop making things up Kirchoff never said. That is NOT in any of Kirchoff's laws.

VernerHornung wrote:
Into the Night wrote:
According to Kirchoff's law,

Kirchoff's law for radiation was derived from a radiative equilibrium between two emitting and absorbing infinite parallel plates
So Verner, ITN actually believes that when two bodies are in thermal equilibrium there IS NO absorption! Just to deepen the nuttyness (sorry I know that helps no one, least of all ITN).
Into the Night wrote:
Nothing that is warmer than something else will absorb any radiance.
IBD thinks this too! link Flys in the face of 239 years of Thermodynamic research but question is how can you even have a rational discussion of Kirchoff given this nuttiness?

VernerHornung wrote:
Into the Night wrote:
Pressure does NOT change absorption spectra at all.

You've made that claim twice and I've replied citing this source twice.... you're gonna repeat your absurdities like a broken record until they become true—a tactic the radicals use to get the public to accept their claims about society. Interesting how little the far right and the far left differ from one another,...
Excellent point! In every case an enemy of science.
VernerHornung wrote:
Into the Night wrote:You are now attempting to misuse Wien's law.
Then how do you explain this?
ITN never explains anything.
VernerHornung wrote:
The Earth is not a closed system. And you can draw a boundary around a system wherever you please. There are no preferred systems, nor closed ones save the universe itself.
As usual very well put. One thing that is abundantly clear on this issue is people preference to focus on just the ground as being "Earth" is based only on our habitat. Equally stupid is to pretend that the ground level can be disregarded and it doesn't matter what happens there as long as the "whole" is doing it's thing.

VernerHornung wrote:
IBdaMann wrote:
The amount of energy absorbed must equal the amount of energy radiated away. All of it.
Radiative balance with the sun for the entire planet need hold only at the top of the atmosphere, because that's really where Earth ends and space begins.
And that's what you could most accurately call the "Surface" right? The mix of all the bits of gas that are the last ones to touch the ball as our outgoing radiance heads off into the void of space. My understanding is that while some reflected, albedo, light makes its way directly from the ground level to space (giving us a nice picture of Earth from up there) almost none of the infra-red radiance we're talking about does? It's all handed off to gas molecules prior to it's final exit?

Also wouldn't it be fair to say this: The "surface" of Earth that absorbs the Suns energy need not be the same "surface" of Earth that radiates it back out into space.
?
Because the ground DOES absorb a great deal of the energy directly but emits almost none of it directly.

VernerHornung wrote:
IBdaMann wrote:
Explain how the addition of a gas,...
If you just add gas, no temperature hike. You need a star, and a gas transparent to the bulk of the star's light but opaque to a portion of the planet's return radiation.
I'm having doubts about that. I'm wondering if you had heavy atmosphere of a gas entirely transparent, or entirely opaque to incoming radiance, what you'd get for mean temp. It seems the thermal energy would still get "bogged down" in that soup. The ocean is a thermal battery of sorts right? Wouldn't a heavy opaque atmosphere do the same thing the ocean does?

IBdaMann wrote:
What you need to explain is how the earth is able to somehow destroy or create energy
This simple denial that the Earth is warmer than it would be without an atmosphere should be stated as such. You pretend that we know nothing about Venus, that the Earth is not warmer at ground level than the moon, and in short, that we know nothing. Except of course that you know you're right.

"Good tests kill flawed theories; we remain alive to guess again." - Karl Popper
ITN/IBD Fraud exposed:  The 2nd LTD add on claiming radiance from cooler bodies can't be absorbed Max Planck debunks, they can't explain:net-thermal-radiation-you-in-a-room-as-a-reference & Proof: no data is ever valid for them
06-10-2019 09:18
Into the NightProfile picture★★★★★
(21596)
tmiddles wrote:
VernerHornung wrote: So we redo the calculation with an adjustment for this:

Earth: (0.70 x 340/5.67 x 10^(-8))^1/4 = 255K
Mars: (0.75 x 147/5.67 x 10^(-8)))^1/4 = 210K

These match the "predicted temperatures" in the planet chart within a margin of rounding error.
Just gotta say nicely done!

VernerHornung wrote:
Mars has more greenhouse gases total overhead, per unit of surface area, than Earth does!
This really seems to corroborate Huffman's theory that it's all about total mass. As well as Blair McDonald here saying that the other gasses are also producing a greenhouse effect: So the "non-greenhouse" gases are also greenhouse gases to some degree

VernerHornung wrote:
The heating of air by sun warms the surface in an indirect way, and should do so without any greenhouse gases present.
It seem pretty direct to me. If the air is hotter then the ground will be too right?

VernerHornung wrote:Nobody in climate change has ever told us why the Martian "greenhouse" is only 6˚C with more gases than we have, and Earth's is 33˚.
I wonder if anyone has modeled it?

I'm still frustrated that you can't do lab experiments to demonstrate this stuff. Forget about finding out why any planet is doing something. Just predict and model Plexiglas boxes full of precise gas mixes with black paper a the back. I'm guessing someone is doing this type of work.

Harry C wrote:...My problem with it all is that you dismissed the science and are using anecdotal correlations to make a case for something you admit that you don't understand. What's the point? To obfuscate or convince yourself?
What do you mean "dismissed the science"? Verner and I are wondering about something. Why is it that way? We don't know. That's OK. It's SCIENCE : D

Nope. Science is a set of falsifiable theories, not contrived data and assumptions.
tmiddles wrote:
This stuff isn't figured out yet as far as we know. I would love to know more! How about you?

IBdaMann wrote:...you are violating thermodynamics...
IBD, who learned his private thermodynamics from a guy on the street in the oral tradition of Physics he adheres too. No book, no research, just word o' mouth.
IBdaMann wrote:
tmiddles wrote:...How did you learn the Stefan-Boltzmann Law ...
Someone explained the science to me,...
In 5 years never a single calculation done.

RQAA. RDCF.
tmiddles wrote:
IBdaMann wrote:
VernerHornung wrote: I've stated over and over again that the energy comes from the sun.
... violate the 1st law of thermodynamics? The earth simply does not have the power to either create or destroy energy.
You simply deny that the Earth is 30 degrees warmer than it should be. Oh no we don't know the temp of Earth or of anything they cry! We can't talk about it they cry! Lame lame lame.

No explanation for you guys at all. Just BS.

RQAA. RDCF.

You just keep asking the same questions that have already been answered.
You just keep making distortions and contextomies.

Repetition fallacy. Argument of the stone fallacy. Argument from randU fallacy. Contextomy fallacy. Redefinition fallacy (random numbers<->science).


The Parrot Killer

Debunked in my sig. - tmiddles

Google keeps track of paranoid talk and i'm not on their list. I've been evaluated and certified. - keepit

nuclear powered ships do not require nuclear fuel. - Swan

While it is true that fossils do not burn it is also true that fossil fuels burn very well - Swan
06-10-2019 10:02
Into the NightProfile picture★★★★★
(21596)
tmiddles wrote:
VernerHornung wrote:
Harry C wrote:
What's the point? To obfuscate or convince yourself?

I'm not trying to "convince" myself of anything. I'm still agnostic regarding details of what causes climate change on a planet, of a dim view toward the environmental scare tactics swirling 'round industrial CO2 on Earth.

I concur and have a similar position at this point. One trap extremists fall into, both the ITN/IBDs of the world and the crying sky is falling environmental doomsdayers, is that they over reach with their claims and end up in an argument where they are dead wrong.

Void argument fallacy. What claims are you talking about? Bulverism fallacy.
tmiddles wrote:
Something can be TRUE and yet be INSIGNIFICANT. If you claim it doesn't exist, and it does, you are doing a great job of creating the illusion that it's VERY SIGNIFICANT since you seem so afraid of it.

Void argument fallacy.
tmiddles wrote:
So it goes with made up science arguments from buffoons like ITN/IBD.

Lie. Neither of us have made any science. We did not create the 1st or 2nd laws of thermodynamics, which you deny, or the Stefan-Boltzmann law, which you deny.
tmiddles wrote:
Real skeptical scientists appear behind a cloud of that BS.

Science isn't scientists. It is a set of falsifiable theories. They are not BS.
tmiddles wrote:
A perfect example of confusing "it exists" with "it is important" is with radiation. Radiation is real!

Yes, it is. So?
tmiddles wrote:
When Fukushima happened you bet some of that radiation got into the coast waters near the plant.

Not a lot. It's actually pretty safe there now. While the one of the reactors was destroyed, the surrounding area is quite safe now.
tmiddles wrote:
Some very stupid people said don't go in the pacific ocean on the US west coast, don't eat fish. This is effectively holding up a MORON sign and discrediting anything that ever comes out of your mouth on the subject again. There is NO upside for anyone when this kind of human stupidity flourishes.

Bulverism fallacy.
tmiddles wrote:
Yes having the other planets seems like our best resource in being able to figure out how it all works. They are real, full, equally complex models we can study. And for what it's worth Verner I'm understanding the basic science a lot better thanks to your work.

Void argument fallacy. 'Greenhouse effect' is the same on all planets: zero.
* You cannot create energy out of nothing.
tmiddles wrote:
Also Harry this was actually "Step one" in the whole science of temperature and atmosphere. 1827 Fourier indicated the similarity between what happens under the glass of a greenhouse and how heat is absorbed in the atmosphere. This led to the term "greenhouse effect.".

Contextomy fallacy. Stop misquoting the great work of scientists like Fourier.
tmiddles wrote:

Fourier noticed that the Earth was warmer that is should be.

No one knows what it should be. The emissivity of Earth is unknown.
tmiddles wrote:
He didn't have the direct measurements from Venus and Mars but he was still able to realize there was an odd extra 30 degrees to our mean temperature at ground level.

No one knows what it should be. The emissivity of Venus and of Mars is unknown.
tmiddles wrote:
VernerHornung wrote:
IBdaMann wrote:....violating Stefan-Boltzmann...
...you explain the correct method for applying it to a body enclosed by a partially transparent shell.
Don't hold your breath Verner. IBD has never explained how to do anything in 5 years.

Lie. RDCF. RQAA.
tmiddles wrote:
IBdaMann wrote:Explain how the addition of a gas, not any additional energy, increases a body's average temperature.
What was your theory on Venus? Oh right, nothing, nothing at all.

None needed. You are just making up numbers again. Argument from randU fallacy.
tmiddles wrote:
IBdaMann wrote:
Venus' atmosphere is irrelevant.


Into the Night wrote:
VernerHornung wrote:
It is a composite system with parts...
According to Kirchoff's law, ...You can't move the goalposts around by separating out parts and then considering them as the whole and back again.
This is completely unfounded. You can consider everything in the universe to be one "part" and you can consider a single atom to be one "part". Stop making things up Kirchoff never said. That is NOT in any of Kirchoff's laws.


Yes he did.
* You can't consider a part of a system the equivalent of any system.
* All systems in thermodynamics must be closed. You can't just change the boundaries of a system on the fly. They must be consistent.
tmiddles wrote:
VernerHornung wrote:
Into the Night wrote:
According to Kirchoff's law,

Kirchoff's law for radiation was derived from a radiative equilibrium between two emitting and absorbing infinite parallel plates
So Verner, ITN actually believes that when two bodies are in thermal equilibrium there IS NO absorption! Just to deepen the nuttyness (sorry I know that helps no one, least of all ITN).

Contextomy fallacy. RDCF. FQAA.
* You can't create energy out of nothing.
* You can't trap or store heat.
* You can't reduce entropy in any system.
* You can't warm a warmer body with a colder one.
Into the Night wrote:
Nothing that is warmer than something else will absorb any radiance.
[/url]IBD thinks this too! link Flys in the face of 239 years of Thermodynamic research but question is how can you even have a rational discussion of Kirchoff given this nuttiness?
[/quote]
* You cannot change the boundaries of any system in thermodynamics on the fly. The system must be closed (consistent).
* You cannot treat any part of a system as the whole system.
tmiddles wrote:
VernerHornung wrote:
Into the Night wrote:
Pressure does NOT change absorption spectra at all.

You've made that claim twice and I've replied citing this source twice.... you're gonna repeat your absurdities like a broken record until they become true—a tactic the radicals use to get the public to accept their claims about society. Interesting how little the far right and the far left differ from one another,...
Excellent point! In every case an enemy of science.

Inversion fallacy. Argument of the stone fallacy. It is YOU that is denying the 1st and 2nd laws of thermodynamics and the Stefan-Boltzmann law. It is YOU that is ignoring the science.
tmiddles wrote:
VernerHornung wrote:
Into the Night wrote:You are now attempting to misuse Wien's law.
Then how do you explain this?
ITN never explains anything.

Already have. RQAA. RDCF. Lie.
tmiddles wrote:
VernerHornung wrote:
The Earth is not a closed system. And you can draw a boundary around a system wherever you please. There are no preferred systems, nor closed ones save the universe itself.
As usual very well put. One thing that is abundantly clear on this issue is people preference to focus on just the ground as being "Earth" is based only on our habitat. Equally stupid is to pretend that the ground level can be disregarded and it doesn't matter what happens there as long as the "whole" is doing it's thing.

Redefinition fallacy (bound system<->closed system).
* You cannot reduce entropy in any system.
* You cannot heat a warmer body using a colder one in any system.
* You cannot create energy out of nothing.
* You cannot destroy energy into nothing.
* You cannot treat a part of a system as the whole system.
tmiddles wrote:
VernerHornung wrote:
IBdaMann wrote:
The amount of energy absorbed must equal the amount of energy radiated away. All of it.
Radiative balance with the sun for the entire planet need hold only at the top of the atmosphere, because that's really where Earth ends and space begins.
And that's what you could most accurately call the "Surface" right?

WRONG. Most of the radiance of Earth is from the surface itself. There is no barrier in the atmosphere. There is no specific top of the atmosphere.
* You cannot trap light.
* You cannot reduce entropy in any system.
tmiddles wrote:
Also wouldn't it be fair to say this: The "surface" of Earth that absorbs the Suns energy need not be the same "surface" of Earth that radiates it back out into space.
?
Because the ground DOES absorb a great deal of the energy directly but emits almost none of it directly.

Paradox. You have taken away the light that is supposedly causing 'global warming'.

Which is it, dude?
tmiddles wrote:
VernerHornung wrote:
[quote]IBdaMann wrote:
Explain how the addition of a gas,...
If you just add gas, no temperature hike. You need a star, and a gas transparent to the bulk of the star's light but opaque to a portion of the planet's return radiation.
I'm having doubts about that. I'm wondering if you had heavy atmosphere of a gas entirely transparent, or entirely opaque to incoming radiance, what you'd get for mean temp. It seems the thermal energy would still get "bogged down" in that soup. The ocean is a thermal battery of sorts right? Wouldn't a heavy opaque atmosphere do the same thing the ocean does?
No. Specific heat is not the same between the two materials. Specific heat does not increase or decrease heat.
* You cannot trap or slow heat.
* You cannot trap light.
* You cannot trap thermal energy. There is always heat.
* You cannot decrease entropy in any system.
tmiddles wrote:
IBdaMann wrote:
What you need to explain is how the earth is able to somehow destroy or create energy
This simple denial that the Earth is warmer than it would be without an atmosphere should be stated as such.

* The emissivity of Earth is unknown. You don't know what the temperature should be.
tmiddles wrote:
You pretend that we know nothing about Venus,

RDCF. Compositional error fallacy. Lie.
tmiddles wrote:
that the Earth is not warmer at ground level than the moon,

It isn't.
tmiddles wrote:
and in short, that we know nothing.

Lie. Compositional error fallacy. RDCF.
tmiddles wrote:
Except of course that you know you're right.

Bulverism fallacy.


The Parrot Killer

Debunked in my sig. - tmiddles

Google keeps track of paranoid talk and i'm not on their list. I've been evaluated and certified. - keepit

nuclear powered ships do not require nuclear fuel. - Swan

While it is true that fossils do not burn it is also true that fossil fuels burn very well - Swan
Edited on 06-10-2019 10:06
06-10-2019 10:10
VernerHornungProfile picture★☆☆☆☆
(133)
IBdaMann wrote:
What you need to explain is how the earth is able to somehow destroy or create energy...

I've done enough explaining. Insulation slows heat loss from a warm body to a cold environment. Whether the insulation blocks convection or radiation doesn't matter; a winter coat does in fact block radiation—in addition to conduction & convection—from the person wearing it. No one's claimed that insulation violates conservation of energy. The only reason pundits can get away with denying the obvious in greenhouse effects is that the mechanism for insulating the Earth is complex and difficult to reproduce on a tabletop.

I'm fully aware of the defective Bill Nye-style experiments with CO2 in bottles where the CO2 is heated directly by the lamp. He'd need an electric arc shining only visible and near-IR light up to λ = 3μm onto a black-painted metal plate suspended inside an insulated thermos bottle with a window for the light to come in. Problem being the apparatus wouldn't look so impressive to naïve viewers.

Nothing that might be staged will convince those who've made up their mind anyway. Los Alamos didn't test the Hiroshima "Little Boy" device, either, as they knew it would work even though it was the first bomb of its kind.

IBdaMann wrote:
Let's stay focused on global "climate." What is that EXACTLY?

I can define what it's not. Here we are on this little blue gem surrounded by dead planets. Seems like a good idea to take care of this home and understand the risks our activities pose to its health. Postulating endless petroleum after industry geologists have told us it's a finite resource, or that the composition of the air doesn't affect Earth's surface temps after the weather and climate bureaus have concluded it does, is foolish. I rarely quiz my auto mechanic on stuff he tells me is wrong with my car.

IBdaMann wrote:
... but your single biggest problem is that this doesn't bother you.

You don't suppose it does? If the uncertainties and what I see as an overestimate of scientists' ability to predict how Earthly climates will evolve left me unfazed, I'd post ads for Citizens Climate Lobby here and email my Congressman to tell him go ahead and wreck the US economy in a panic. I'm looking for policies that address fossil fuels supply limitations and pollution risks in a measured fashion, perhaps starting with electric power since converting vehicles still awaits proper technology. But the eggplant heads are gonna hand the ball to AOC.

tmiddles wrote:
It seem pretty direct to me. If the air is hotter then the ground will be too right?

I dunno. The air can heat the ground directly only if it's hotter than the ground, and any energy from the sun that goes into heating the air won't go into heating the surface. Hot as Venus's air is, it's still cooler than the surface. It does seem, however, that N2 and O2 help keep the Earth warm. If all we had was our H2O and CO2, we'd possess an atmosphere almost as thin as Mars'. Hard to imagine it balmy.

N2 acts as an insulator for objects that lose heat by conduction, as it's the "active ingredient" in fiberglass insulation for houses. But planets lose heat by radiation since the vacuum of outer space doesn't conduct or convey. A major fly in the soup is that none of these things are blackbodies, and the gases exist as a mixture with its own properties.

IBdaMann wrote:
I wonder if anyone has modeled it?...Forget about finding out why any planet is doing something. Just predict and model Plexiglas boxes full of precise gas mixes with black paper at the back. I'm guessing someone is doing this type of work.

Basically what I'm thinking of, though we'll want better insulation than Plexiglas to be able to detect heating of surfaces induced by visible light before it's conducted away. We don't notice heat coming off LEDs and fluorescent lights; only an intense source like the sun gives off enough in the visible band for rapid heating.

Funny thing is I'm not sure a small-scale model would tell us much. We already know that the radiative greenhouse effect is real, physically, in just the way Oppenheimer & Feynman knew explosive fission in U235 was real having never detonated any of it. The planet is the problem, as scaling something up often results in surprises. Toy boats in bathtubs don't reproduce the behavior of ocean liners beyond the principle of buoyancy, and a lab greenhouse that works fine may not work, or operate differently than expected, if you make it as big as a planet.

Although I've no idea how N2 would operate as an insulator for a radiating planet, I'm not counting it out yet. There's always physics we haven't discovered, and scale matters. I think it unwise to assume an ocean of N2 weighing quadrillions of tons must act the same way the amount in a 2-liter bottle does when it comes to a global climate system. Saturn's moon Titan is an example of a celestial body with a dense N2 atmosphere, and I hope we send more probes to study it.


Never try to solve an NP-complete problem on your own with pencil & paper.
06-10-2019 12:16
tmiddlesProfile picture★★★★★
(3979)
Into the Night wrote:...fallacy.
Sums you up.

VernerHornung wrote:
tmiddles wrote:
It seem pretty direct to me. If the air is hotter then the ground will be too right?
I dunno. The air can heat the ground directly only if it's hotter than the ground,
But it will result in the ground being hotter just by being "in the way". Insulation. I guess that's not considered direct?

VernerHornung wrote:
Hot as Venus's air is, it's still cooler than the surface.
Do we actually know that? I would imagine that there was certainly a thermometer extended from the probe but would they have actually taken a contact measurement from the rock or dirt below?

VernerHornung wrote:
It does seem, however, that N2 and O2 help keep the Earth warm.
At the very least they are able to possess thermal energy. This means that they can participate in adding to the maze incoming energy must traverse as it passes through the Earth system before it becomes outgoing energy.

Here's a basic couple questions for alternate Earth bodies:
With no atmosphere just a simple thin hollow shell of a ball (not much matter) vs. a solid ball (lots of matter). Doesn't the solid ball have a higher night time temperature? But matter has a draw back, it can't flow or convect or radiate within itself, only conduct. So for a 3rd version let's do the solid ball covered with swirling liquid, no atmosphere, and then as a 4th version add a heavy, thick dust cloud, opaque to light. Are we increasing the amount of thermal energy winding it's way through the maze?

I see a denser, more complex and dynamic system resulting in a longer and more complex path for energy to traverse before it is able to leave. That means more thermal energy is present.

It's like trying to leave Ikea.

VernerHornung wrote:
IBdaMann wrote: OOPS VERNER THIS WAS ME NOT IBD
I wonder if anyone has modeled it?...
Funny thing is I'm not sure a small-scale model would tell us much. ...Toy boats in bathtubs don't reproduce the behavior of ocean liners beyond the principle of buoyancy,...
What I'm saying is that in a world where some people, including the US president, deny that buoyancy exists, it is worth demonstrating!

"Good tests kill flawed theories; we remain alive to guess again." - Karl Popper
ITN/IBD Fraud exposed:  The 2nd LTD add on claiming radiance from cooler bodies can't be absorbed Max Planck debunks, they can't explain:net-thermal-radiation-you-in-a-room-as-a-reference & Proof: no data is ever valid for them
Edited on 06-10-2019 12:22
06-10-2019 22:46
Into the NightProfile picture★★★★★
(21596)
VernerHornung wrote:
IBdaMann wrote:
What you need to explain is how the earth is able to somehow destroy or create energy...

I've done enough explaining. Insulation slows heat loss from a warm body to a cold environment.

* It is not possible to trap or slow heat. Insulation reduces heat. Putting a coat on a rock does not make the rock warm.
VernerHornung wrote:
Whether the insulation blocks convection or radiation doesn't matter; a winter coat does in fact block radiation—in addition to conduction & convection—from the person wearing it. No one's claimed that insulation violates conservation of energy.

You do, then you don't. You are being irrational. You must clear your paradox.
VernerHornung wrote:
The only reason pundits can get away with denying the obvious in greenhouse effects is that the mechanism for insulating the Earth is complex and difficult to reproduce on a tabletop.

There is no 'greenhouse effect'. There is nothing to reproduce. No gas or vapor is capable of warming the Earth using emitted IR from Earth's surface.
VernerHornung wrote:
I'm fully aware of the defective Bill Nye-style experiments with CO2 in bottles where the CO2 is heated directly by the lamp. He'd need an electric arc shining only visible and near-IR light up to λ = 3μm onto a black-painted metal plate suspended inside an insulated thermos bottle with a window for the light to come in. Problem being the apparatus wouldn't look so impressive to naïve viewers.

Absorption of surface IR by any gas or vapor in the atmosphere does not warm the Earth or the surface.
VernerHornung wrote:
Nothing that might be staged will convince those who've made up their mind anyway. Los Alamos didn't test the Hiroshima "Little Boy" device, either, as they knew it would work even though it was the first bomb of its kind.

WRONG. Little Boy was tested at Los Alamos, NM. That device was later dropped on Nagasaki, not Hiroshima. Fat Man was tested at Sorocco, NM. That device was used on Hiroshima.
VernerHornung wrote:
IBdaMann wrote:
Let's stay focused on global "climate." What is that EXACTLY?

I can define what it's not. Here we are on this little blue gem surrounded by dead planets. Seems like a good idea to take care of this home and understand the risks our activities pose to its health. Postulating endless petroleum after industry geologists have told us it's a finite resource, or that the composition of the air doesn't affect Earth's surface temps after the weather and climate bureaus have concluded it does, is foolish. I rarely quiz my auto mechanic on stuff he tells me is wrong with my car.

So you can't define 'global climate'. BTW, I always quiz an auto mechanic working on my car. I usually just fix my own car. I'll quiz doctors and lawyers too. It's only prudent. NOTHING about their degree or license makes them right. I've seen airline pilots do stupid things too. I'll mention it when they do. I've seen air traffic controllers make some pretty egregious errors. I'll override them when they do when I'm flying. Airline pilots will do the same thing.
VernerHornung wrote:
IBdaMann wrote:
... but your single biggest problem is that this doesn't bother you.

You don't suppose it does? If the uncertainties and what I see as an overestimate of scientists' ability to predict how Earthly climates will evolve left me unfazed, I'd post ads for Citizens Climate Lobby here and email my Congressman to tell him go ahead and wreck the US economy in a panic. I'm looking for policies that address fossil fuels supply limitations and pollution risks in a measured fashion, perhaps starting with electric power since converting vehicles still awaits proper technology. But the eggplant heads are gonna hand the ball to AOC.

Oil is renewable. Natural gas is renewable. Coal is cheap. There is plenty of it. Market prices determine which energy source to use, no government is needed.
VernerHornung wrote:
tmiddles wrote:
It seem pretty direct to me. If the air is hotter then the ground will be too right?

I dunno. The air can heat the ground directly only if it's hotter than the ground, and any energy from the sun that goes into heating the air won't go into heating the surface. Hot as Venus's air is, it's still cooler than the surface. It does seem, however, that N2 and O2 help keep the Earth warm. If all we had was our H2O and CO2, we'd possess an atmosphere almost as thin as Mars'. Hard to imagine it balmy.

N2 acts as an insulator for objects that lose heat by conduction, as it's the "active ingredient" in fiberglass insulation for houses.

No gas or vapor is a thermal insulator. N2 is no different. N2 used in doubled window panes is trapped. Convective heating is reduced. N2 conducts heat about the same as most other gases in the atmosphere. CO2 conducts heat BETTER than most any other gas in the atmosphere.
VernerHornung wrote:
But planets lose heat by radiation since the vacuum of outer space doesn't conduct or convey. A major fly in the soup is that none of these things are blackbodies, and the gases exist as a mixture with its own properties.

All bodies emit black body radiance. You are denying the Stefan-Boltzmann law again.
VernerHornung wrote:
IBdaMann wrote:
I wonder if anyone has modeled it?...Forget about finding out why any planet is doing something. Just predict and model Plexiglas boxes full of precise gas mixes with black paper at the back. I'm guessing someone is doing this type of work.

Basically what I'm thinking of, though we'll want better insulation than Plexiglas to be able to detect heating of surfaces induced by visible light before it's conducted away. We don't notice heat coming off LEDs and fluorescent lights; only an intense source like the sun gives off enough in the visible band for rapid heating.

No? LED's often require heatsinks. Why do you suppose they do? You are confusing tuned radiance with untuned radiance. Black body radiance is 'untuned' radiance. It is not the only way to generate light.
VernerHornung wrote:
Funny thing is I'm not sure a small-scale model would tell us much. We already know that the radiative greenhouse effect is real, physically,

* No gas or vapor has the capability to warm the Earth using IR emitted from Earth's surface.
VernerHornung wrote:
in just the way Oppenheimer & Feynman knew explosive fission in U235 was real having never detonated any of it.

They didn't know until they detonated it. Both devices were tested before being used in Japan.
VernerHornung wrote:
The planet is the problem, as scaling something up often results in surprises. Toy boats in bathtubs don't reproduce the behavior of ocean liners

Yes they do. Models of a proposed ocean liner tells you a LOT about the full sized ocean liner. That's how people design them. Model airplanes tell you a LOT about the full sized airplane as well. Even the Space Shuttle design was tested using model aircraft first.
VernerHornung wrote:
beyond the principle of buoyancy,

Not just buoyancy, but both profile and dynamic drag, weight and balance (yes, boats have to pay attention to that!), and even top heaviness.
VernerHornung wrote:
and a lab greenhouse that works fine may not work, or operate differently than expected, if you make it as big as a planet.

No difference, except we don't built planet sized greenhouses.
VernerHornung wrote:
Although I've no idea how N2 would operate as an insulator for a radiating planet, I'm not counting it out yet.

It doesn't.
VernerHornung wrote:
There's always physics we haven't discovered, and scale matters.

Scale does not change any law of physics.
VernerHornung wrote:
I think it unwise to assume an ocean of N2 weighing quadrillions of tons must act the same way the amount in a 2-liter bottle does

It behaves in exactly the same way in all respects.
VernerHornung wrote:
when it comes to a global climate system.

Define 'global climate system'. Define 'climate change'.
VernerHornung wrote:
Saturn's moon Titan is an example of a celestial body with a dense N2 atmosphere, and I hope we send more probes to study it.

Why not?


The Parrot Killer

Debunked in my sig. - tmiddles

Google keeps track of paranoid talk and i'm not on their list. I've been evaluated and certified. - keepit

nuclear powered ships do not require nuclear fuel. - Swan

While it is true that fossils do not burn it is also true that fossil fuels burn very well - Swan
06-10-2019 22:48
Into the NightProfile picture★★★★★
(21596)
tmiddles wrote:
Into the Night wrote:...fallacy.
Sums you up.

VernerHornung wrote:
tmiddles wrote:
It seem pretty direct to me. If the air is hotter then the ground will be too right?
I dunno. The air can heat the ground directly only if it's hotter than the ground,
But it will result in the ground being hotter just by being "in the way". Insulation. I guess that's not considered direct?

VernerHornung wrote:
Hot as Venus's air is, it's still cooler than the surface.
Do we actually know that? I would imagine that there was certainly a thermometer extended from the probe but would they have actually taken a contact measurement from the rock or dirt below?

VernerHornung wrote:
It does seem, however, that N2 and O2 help keep the Earth warm.
At the very least they are able to possess thermal energy. This means that they can participate in adding to the maze incoming energy must traverse as it passes through the Earth system before it becomes outgoing energy.

Here's a basic couple questions for alternate Earth bodies:
With no atmosphere just a simple thin hollow shell of a ball (not much matter) vs. a solid ball (lots of matter). Doesn't the solid ball have a higher night time temperature? But matter has a draw back, it can't flow or convect or radiate within itself, only conduct. So for a 3rd version let's do the solid ball covered with swirling liquid, no atmosphere, and then as a 4th version add a heavy, thick dust cloud, opaque to light. Are we increasing the amount of thermal energy winding it's way through the maze?

I see a denser, more complex and dynamic system resulting in a longer and more complex path for energy to traverse before it is able to leave. That means more thermal energy is present.

It's like trying to leave Ikea.

VernerHornung wrote:
IBdaMann wrote: OOPS VERNER THIS WAS ME NOT IBD
I wonder if anyone has modeled it?...
Funny thing is I'm not sure a small-scale model would tell us much. ...Toy boats in bathtubs don't reproduce the behavior of ocean liners beyond the principle of buoyancy,...
What I'm saying is that in a world where some people, including the US president, deny that buoyancy exists, it is worth demonstrating!



There is no 'maze'.
* It is not possible to trap or slow heat.
* It is not possible to trap light.
* It is not possible to create energy out of nothing.


The Parrot Killer

Debunked in my sig. - tmiddles

Google keeps track of paranoid talk and i'm not on their list. I've been evaluated and certified. - keepit

nuclear powered ships do not require nuclear fuel. - Swan

While it is true that fossils do not burn it is also true that fossil fuels burn very well - Swan
07-10-2019 05:43
IBdaMannProfile picture★★★★★
(14404)
VernerHornung wrote:I've done enough explaining.

That's convenient.

Well, for the present, everything you have written thus far is summarily dismissed. Let me know when you can explain what I have requested.

VernerHornung wrote: Insulation slows heat loss from a warm body to a cold environment.

The earth has no insulation.


VernerHornung wrote:
IBdaMann wrote:
Let's stay focused on global "climate." What is that EXACTLY?

I can define what it's not.

So can I. It's not peanut butter.

Now, how about defining what it is exactly and unambiguously ...

VernerHornung wrote: Here we are on this little blue gem surrounded by dead planets. Seems like a good idea to take care of this home and understand the risks our activities pose to its health.

Great idea. I understand the risks.

Now what?

VernerHornung wrote: Postulating endless petroleum after industry geologists have told us it's a finite resource,

Instead of pretending to speak for countless, unnamed individuals who are not present to be cross-examined, explain how deeply oil wells extend and whether you believe heat and pressure were needed for the hydrocarbons to form.

VernerHornung wrote: ... or that the composition of the air doesn't affect Earth's surface temps after the weather and climate bureaus have concluded it does, is foolish.

Instead of implying that I am foolish for daring to question the countless, unspecified bureaus and people for whom you pretend to speak that coincidentally are not present to be cross-examined, explain how you believe that any substance has the magickal superpower to violate thermodynamics and Stefan-Boltzmann.

VernerHornung wrote: I rarely quiz my auto mechanic on stuff he tells me is wrong with my car.

You don't question anything that you are ordered to believe by the people who are reaming misinformation into you.

IBdaMann wrote: I'm looking for policies that address fossil fuels supply limitations and pollution risks

Explain these "supply limitations" in more detail. You haven't given any clue as to why these are of any concern.

Do you consider CO2 to be "pollution"?


.


I don't think i can [define it]. I just kind of get a feel for the phrase. - keepit

A Spaghetti strainer with the faucet running, retains water- tmiddles

Clouds don't trap heat. Clouds block cold. - Spongy Iris

Printing dollars to pay debt doesn't increase the number of dollars. - keepit

If Venus were a black body it would have a much much lower temperature than what we found there.- tmiddles

Ah the "Valid Data" myth of ITN/IBD. - tmiddles

Ceist - I couldn't agree with you more. But when money and religion are involved, and there are people who value them above all else, then the lies begin. - trafn

You are completely misunderstanding their use of the word "accumulation"! - Climate Scientist.

The Stefan-Boltzman equation doesn't come up with the correct temperature if greenhouse gases are not considered - Hank

:*sigh* Not the "raw data" crap. - Leafsdude

IB STILL hasn't explained what Planck's Law means. Just more hand waving that it applies to everything and more asserting that the greenhouse effect 'violates' it.- Ceist
07-10-2019 07:05
tmiddlesProfile picture★★★★★
(3979)
Into the Night wrote:There is no 'maze'.
You have yet to present anything but platitudes in 5 years. Waste of time.

IBdaMann wrote:
Do you consider CO2 to be "pollution"?

Do you consider phosphorus in the water to be pollution? I would say sometimes it is.

CO2 might be pollution if there is enough of it to cause a problem.

A ecosystem has a balance and of course you can mess it up. We couldn't even get Biosphere to work so don't assume it's easy or resilient.


"Good tests kill flawed theories; we remain alive to guess again." - Karl Popper
ITN/IBD Fraud exposed:  The 2nd LTD add on claiming radiance from cooler bodies can't be absorbed Max Planck debunks, they can't explain:net-thermal-radiation-you-in-a-room-as-a-reference & Proof: no data is ever valid for them
Edited on 07-10-2019 07:06
07-10-2019 19:07
Into the NightProfile picture★★★★★
(21596)
tmiddles wrote:
Into the Night wrote:There is no 'maze'.
You have yet to present anything but platitudes in 5 years. Waste of time.
tmiddles wrote:
[quote]IBdaMann wrote:
Do you consider CO2 to be "pollution"?

Do you consider phosphorus in the water to be pollution?

No.
tmiddles wrote:
I would say sometimes it is.

And this brings up the old question which idiots like you never seem to be able to answer:
What is 'polllution'? Define it.
tmiddles wrote:
CO2 might be pollution if there is enough of it to cause a problem.

Such as? Void argument.
tmiddles wrote:
A ecosystem has a balance and of course you can mess it up. We couldn't even get Biosphere to work so don't assume it's easy or resilient.

What does biosphere2 have to do with anything on the Earth? Define 'ecosystem'. This is a buzzword from the Church of Green. It's meaningless.


The Parrot Killer

Debunked in my sig. - tmiddles

Google keeps track of paranoid talk and i'm not on their list. I've been evaluated and certified. - keepit

nuclear powered ships do not require nuclear fuel. - Swan

While it is true that fossils do not burn it is also true that fossil fuels burn very well - Swan
07-10-2019 21:41
VernerHornungProfile picture★☆☆☆☆
(133)
tmiddles wrote:Do we actually know that? I would imagine that there was certainly a thermometer extended from the probe but would they have actually taken a contact measurement from the rock or dirt below?

The thermometer was at some height within 3 ft of the surface. Veneras 13 & 14 had soil penetration probes but these measured hardness & electrical conductivity only. The rocks might have been several degrees warmer than the 462˚C reported for near-surface air. This difference is why the NWS prescribes thermometers to be six feet (2 m) above ground in the US.

Reason to suspect solid surface hotter than air is that there's a wind at the surface, about 2 mph but with the high air pressure enough to transport dust. (You can see where dirt has collected between rock slabs in the photos.) Winds result from convection, rising air. It's clear from Venera data that the clouds are heated by incoming sunlight, and winds aloft are very strong, circling the planet in 4 Earth days, so there's more convection driven by the sun at the altitudes Huffman talks about.



Heating of upper air & clouds by sun marked by red arrow. The jag in the curve is a temperature inversion where air is stagnant, but strong winds blow just below it, in the cloud top zone.

tmiddles wrote:
But it will result in the ground being hotter just by being "in the way". Insulation. I guess that's not considered direct?

It's "in the way," but in in a funny way. The NWS has a website, Jet Stream, that explains ordinary weather like cold fronts without too much math. This site shows how air warmed by surface becomes buoyant, rises, expands and cools. It relates to an earlier equation I posted, conservation of energy or 1st law of thermodynamics.

ΔU = Q — W

The heat Q increases the internal energy U of the air, but the air does work W while expanding, which diminishes this rise. The work is governed by the ideal gas law

PV = nRT, where R ≈ 8.3 Joules per mole per Kelvin,

Increments of the work, called "pressure-volume work," are

ΔW = P ΔV,

where ΔV is the volume change from expansion. You feel that work trying to inflate a car tire with a hand pump. Most of the work goes into expanding the tire at low pressure at first, and when the tire's maximum size, its air gets hotter as you continue pumping to raise the pressure. This brings up constant-volume work,

ΔW = V ΔP

done by winds. (Since you're doing work on the air with a tire pump instead of adding heat, the Q is negative so the ΔU is positive unless heat leaves the system.) Air also has enthalpy changes because it has a heat capacity and because water (Earth) or sulfuric acid (Venus) evaporate and condense in the atmosphere. The general formulas for enthalpy H in ideal gas are

H = U + PV and
ΔH = ΔQ = Cp ΔT

if constant pressure and no evaporation or condensation, where Cp is the heat capacity of the gas at constant pressure. (Oh, my.) Beginning from these, a set of barometric equations can be derived. Without going into details, we note that the temperature drops linearly with altitude while the pressure drops exponentially. On a dry day, the temperature drop, called the adiabatic lapse rate, is about 10˚C or 10K per kilometer of ascent in a plane, and the pressure drops so that ½ the sea level value is reached at 6 km altitude and ¼ of sea level at 12 km.

At low elevations the drop works out to 3 millibars every 100 feet, so that air pressures in Utah's valleys are around 15% lower than in Florida.

tmiddles wrote:
With no atmosphere just a simple thin hollow shell of a ball (not much matter) vs. a solid ball (lots of matter). Doesn't the solid ball have a higher night time temperature?

Heat loss from a solid ball will be slower than from a thin shell, in fact so much slower the Earth's core is as hot as the sun's surface but almost none of the interior heat gets to the surface on human lifetime scales. Out of the 340 W/m^2 heating we've discussed, just a few milliwatts comes from below ground.

tmiddles wrote:
...So for a 3rd version let's do the solid ball covered with swirling liquid, no atmosphere...

Any fluid will operate to "store" energy in kinetic form as winds or ocean currents, or as the pressure-volume work done by air, a compressible fluid. These forms of energy eventually dissipate to heat through friction, yet there's a time delay involved. Winds run like a conveyer belt driven by the sun. Air heated at the equator rises and moves toward the poles at altitude, as if riding the conveyor, and is replaced by cooler air from the poles as if that were coming back along the lower, return side of the belt. The warm air aloft of course cools by radiation as it goes.

What I don't know is whether all this will result in a warmer surface, once thermal equilibrium with rays from the sun is reached. It will delay any warming or cooling, however. Oceans & winds keep Earth from having intolerable extremes with latitude and time of day. The sad part is a lab-sized apparatus with gas above a metal plate being heated by a lamp won't generate much in the way of wind.

The main reason I don't know is because greenhouse gases don't store energy; they absorb and re-radiate almost immediately. While I have doubts that pure, dry, still N2 effects much temp hike relative to vacuum, it increases the absorptivity of H2O and CO2 through spectral line broadening because of its pressure. The N2 is also part of a system with dust & clouds. The latter intercept rays at both visible and IR wavelengths and transfer heat to nearby air by conduction. I simply lack the ability to deal with the complications.

tmiddles wrote:
What I'm saying is that in a world where some people, including the US president, deny that buoyancy exists, it is worth demonstrating!

Trumpon, a ghoulish world where boats can't float but the continents are littered with chunks of coal; oil geysers shoot skyward in an endless hiss of hydrocarbons from the mantle. Yes, we could use a nice experiment. Finding one amid the Google trash is hopeless; I'll probably have to go the college and use its physics search engine—with the librarian's help. A couple downloadable planetary science books are at

Introduction to Planetary Science (elementary)
Gunter Faure & Teresa Mensing
Springer, 2007
http://www.doganaydal.com/nesneler/kutuphanekitaplar/INTRODUCTION_TO_PLANETARY_SCIENCE_THE_GEOLOGICAL_PERSPECTIVE.PDF

Fundamental Planetary Science (advanced)
Jack Lissauer & Imke Pater
Internet Archive
https://archive.org/details/Fundamental_Planetary_Science_Physics_Chemistry_and_Habitability_by_Jack_J._Liss

Both are unfortunately committed to the greenhouse paradigm. They present good evidence for existence and importance of the mechanism, yet seem to discount the possibility of co-occurring factors or gaps in scientific understanding I'm wary of given once upon a time phlogiston ruled heat transfer theory. PC has even managed to demonize a molecule, switching its oxygens from the placid white I remember from chem to a more ominous hue:


Wikimedia CO2
https://commons.wikimedia.org/wiki/File:Carbon_dioxide_3D_spacefill.png

tmiddles wrote: I see a denser, more complex and dynamic system resulting in a longer and more complex path for energy to traverse before it is able to leave.

Climate models skip a lot, including second derivatives in equations, because of the grids they use, and problems inhere in the oft-quoted 33˚C "warming" figure as well. This is based on a reference surface, a gray body of emissivity 0.7, which gives 255K. The moon is actually colder than this despite being much darker than Earth. One can employ the reference as a benchmark against which observations or model values can be pegged to, but it doesn't give us an absolute magnitude for greenhouse warming.



And to think they already projected this stuff 7 years ago. It was the "2˚ of warming" paper, and now we're in "damage control mode" overshooting 1.5˚ in the la-la land of Red CO2. Can we predict our cumulative exhaust fumes for 2200 much less the last degree based on the sketches available? We might be flying interstellar missions in 2200.

Cumulative carbon as a policy framework
H. Damon Matthews et. al.
Royal Society, 2012
https://ia800703.us.archive.org/27/items/pubmed-PMC3405665/PMC3405665-rsta.2012.0064.pdf


Never try to solve an NP-complete problem on your own with pencil & paper.
Edited on 07-10-2019 22:32
Page 3 of 6<12345>>>





Join the debate Do I have the CO2 calamity math right? (help from an expert please):

Remember me

Related content
ThreadsRepliesLast post
Basic Mistakes in Math, Just an FYI, 2 x 2 ≠ 4.027-01-2024 19:01
Math Help5308-12-2023 01:56
Fossil Fuel Substitution for reduced emission of CO2, mercury, lead, arsenic, cadmium..39201-12-2023 21:58
Proof That Too Much CO2 Is An Existential Threat32607-11-2023 19:16
There is no scientific theory or evidence that suggest CO2 traps heat better than O2 or N253330-01-2023 07:22
▲ Top of page
Public Poll
Who is leading the renewable energy race?

US

EU

China

Japan

India

Brazil

Other

Don't know


Thanks for supporting Climate-Debate.com.
Copyright © 2009-2020 Climate-Debate.com | About | Contact